You are on page 1of 200

EMERGENCY

MEDICINE PAPER UPDATED VERSION TILL NOVEMER 2016

40. asprin toxicity with ABG : show low CO2 and low HCO3 ?
with acidic PH ?. cause ?
resp. alkalosis followed by metabolic acidosis
metabolic acidosis followed by resp. alkalosis
metabolic acidosis
Answer: A
Respiratory alkalosis progressing to metabolic acidosis is the key for
aspirin overdose diagnosis. Aspirin interferes with oxidative
phosphorylation increasing lactate levels. Reference: Master the board
page: 604

41. bee sting treatment?


answer: - For local reactions: Provide supplemental oxygen
Diphenhydramine limits the size of the local reaction. Clean the wound
and remove the stinger if present. Apply ice or cool packs. Elevate the
extremity to limit edema. if generalized reactions developed; Treatment
should include an initial intravenous (IV) bolus of 10-20 mL/kg isotonic
crystalloids in addition to diphenhydramine and epinephrine. Reference:
http://emedicine.medscape.com/article/768764-treatment#d9

42. child ingested iron what is treatment:


Answer: - Deferoxamine is the iron-chelating agent of choice.
Deferoxamine binds absorbed iron, and the iron-deferoxamine complex
is excreted in the urine. Reference: Medscape

43. in emergency department pt come with close head truma and loss
of consciousness what is the first thing to do:
intubation and hyperventalition
asses airway
pupalliry responce
glascow coma scale
Answer: B Assess Airway. Establishing and maintaining airway patency
takes precedence over all other treatment. Reference: First Aid Step 2
page 466

DR. IMRAN ASGHAR November 28, 2016


Drimran823@gmail.com
EMERGENCY MEDICINE PAPER UPDATED VERSION TILL NOVEMER 2016

44. An alcoholic comes to you with symptoms of alcohol withdrawal. Last


drink he consumed was 2 days back. What drug will you give?
a. Naloxone
b. Diazepam
c. Nicotine
d. Disulfiram
Answer : B Diazepam. Basic protocol for alcohol withdrawal treatment is
diazepam 20mg PO every 2hrs til regression of severe withdrawal
symptoms + Thiamine 100 mg IM then 100 mg PO OD for 3 days in
addition to hydration.
Reference: Toronto notes.

45. Young female stayed out in sun at 42 degree .. she came to ER


later tired.. muscle cramp.. vitally stable except T: 38 how to manage? A.
Normal sline B. Cold pack
Answer: B
Heat Stroke - Rapid reduction of the core body temperature is the
cornerstone of treatment because the duration of hyperthermia is the
primary determinant of outcome. Once heatstroke is suspected, cooling
must begin immediately and must be continued during the patient's
resuscitation.
Reference: http://emedicine.medscape.com/article/166320-treatment

46. elderly patient had motor vehicle accident, there is problem with
ventilation. what is next step:
exaggerated jaw thrust
more head tilt
intubate immediately
answer : C. Failure of ventilation is an indication for endotracheal
intubation. Reference: Medscape.

47. Dog bite infections?


Gram negative bacillus bacterua
virals
parasite
answer : A. The most common pathogens in dog bites are Pasteurella
spp. (both Pasteurella multocida and Pas-teurella canis),
Staphylococcus and Streptococcus spp., and the fastidious Gram-

DR. IMRAN ASGHAR November 28, 2016


Drimran823@gmail.com
EMERGENCY MEDICINE PAPER UPDATED VERSION TILL NOVEMER 2016

negative rod Capno-cytophaga canimorsus (previously known as the


CDC and Prevention Group Dysgonic Fermenter-2).
Treatment:is amoxicillin/clavulanate plus tetanus vaccination booster if
more 5 years since last injection. Reference: Master the board.
Reference: http://www.medscape.com/viewarticle/739023_4

48. patient presented to ER with history of drug overdose and coma for
the last 8 hours on examina-tion absent gag reflex best management is :
A.iv naloxone
B.gastric lavage
C.immediate endotracheal intubation
D. charcoal
Answer: C
Airway protection – Airway protection by endotracheal intubation should
be performed early in the poi-soned patient with depressed mental
status, unless the cause is easily reversible (eg, opioid intoxication or
hypoglycemia), because of the high risk for aspiration and its associated
complications, particularly when gastric decontamination procedures
need to be undertaken
http://www.uptodate.com/contents/general-approach-to-drug-poisoning-
in-adults

49. case head trauma on parietal lobe subdural hematoma which artery
is injured?
a-superficial temporal .
b- mid cerebral
c- Rt.cerebral
answer: C
Middle cerebral artery supplies parietal lobe. however, subdural
hematomas are most commonly caused by tearing of the bridging veins
that drain from the surface of the brain to the dural sinuses. Ar-terial
rupture can also result in Subdural Hematoma and most of them were
caused by injuries to small cortical arteries. Reference:

DR. IMRAN ASGHAR November 28, 2016


Drimran823@gmail.com
EMERGENCY MEDICINE PAPER UPDATED VERSION TILL NOVEMER 2016

50. Patient is eating at a restaurant with his friend; he choked with a


piece of meat, his friend who was an emergency assistant performed
Heimlich maneuver but without benefit. He then decided to perform a
tracheostomy, during tracheostomy which of the following structures will
be cut?
Cricoid cartilage
Thyroid cartilage
Cricothyroid membrane
Answer: C.

51. young man close the door on his nail color become blue under nail
what will u do ?
evacuate hematoma
remove nail
reassure
Answer: C A painless and small subungual hematoma usually requires
no treatment and its according to the nail edges status. evacuation is
indicated in the presence of a painful subungual hematoma with the nail
edges intact.
Reference: http://www.webmd.com/skin-problems-and-
treatments/bleeding-under-nail?page=2 ,
http://emedicine.medscape.com/article/82926-overview#a2

52. pt involved in MVA the impact especially in forehead examination


forehead laceration & fx ,dis-charge from nose clear positive glucose
test ( I think mean CSF leak ) Which cranial n affected :
Optic
Olfactory
Ophthalmic
Oculomotor
Answer: D
Cerebrospinal fluid rhinorrhea in the acute phase after trauma has been
reported in as many as 39% of the patients with skull base
fractures.[104] Patients present with a variety of symptoms depending
on the acuteness of the event. In the acute phase following the traumatic
event, patients may present with epistaxis, nasal discharge, periorbital
ecchymosis, chemosis, oculomotor impairment, anosmia, motor deficit,
open-head injury with CSF leakage, loss of vision, cranial nerve deficits
(most frequently, first–third and fifth–seventh cranial nerve injuries),
meningitis, and pneumocephalus Reference:
http://www.medscape.com/viewarticle/765460_3

DR. IMRAN ASGHAR November 28, 2016


Drimran823@gmail.com
EMERGENCY MEDICINE PAPER UPDATED VERSION TILL NOVEMER 2016

53. What is the treatment of Aspirin toxicity?


Alkalinization of urine.
Gastric lavage.
Activated charcoal.
Answer: A. Alkalinization of urine. Reference: First Aid step 2 page 482

54. Which of the following substance ingestion is a contraindication to


gastric lavage?
Aspirin.
Benzodiazepine.
Some kind of bleach or cleaning products.
Maybe hydrocarbons.
Answer: C
Caustics ingestion (drain cleaners) is a contraindication to Gastric
lavage.
Reference: Master the board page: 602

55. child girl obese try to suicide and eat alot of drug because of her
friend, came to ER:
immediate referral to psychiatry.
treatment of acute depression.
Answer: A
• ensure patient safety: close observation, remove potentially dangerous
objects from person and room
• assess thoughts (ideation), means, action (preparatory, practice
attempts), previous attempts
• admit if there is evidence of intent and organized plan, access to lethal
means, psychiatric disorder, intoxication (suicidal ideation may resolve
with few days of abstinence)
• patient may require certification if unwilling to stay voluntarily
• do not start long-term medications in the ED
• psychiatry or Crisis Intervention Team consult
Reference: Toronto Notes

DR. IMRAN ASGHAR November 28, 2016


Drimran823@gmail.com
EMERGENCY MEDICINE PAPER UPDATED VERSION TILL NOVEMER 2016

56. Pt with shock & Cherry red skin;


Bacteremia
Septicemia
Carbon monoxidemia
Answer : c

57. Old man came to ER with SOB cough, fever, what is the next step of
management
. a.Put him in negative pressure room
b.Xray
c-Antibiotics
answer: B X- Ray ,cough and fever with respiratory infection order CXR
to notes any interstitial changes .

58. Child on picnic with family, presented to the ER with high suspicion
of foreign body inhalation. The most common location in the chest will be
in?
a- Right Main Bronchus
b- Left Main Bronchus
c- Carina of Trachea
d- Inlet of Larynx

Answer: A Right Main Bronchus 52% of cases


Reference: uptodate

59.Child swallowed battery, now it is lodged in esophagus Ttt ?


a-Endoscopic removal
b-Wait ..
Answer:A Endoscopic removal
Reference: http://www.poison.org/battery/guideline

60. Man have 41 c temp with muscle spasm mangment?


a-core cooling
b- syprying warm
c- warm iv

Answer: A

DR. IMRAN ASGHAR November 28, 2016


Drimran823@gmail.com
EMERGENCY MEDICINE PAPER UPDATED VERSION TILL NOVEMER 2016

61. Healthy baby was in picnic with his family .. Sudenly he get SOB
..come to ER the chest oscultation :RT wheezing in rt hemithorax, What
you will find in the CXR (foreign body),?!

RT lower consolidation
Hyperinflation with midline shift.
RT Atlectasis
ANSWER: B
Children will more often display
signs of air-trapping while adults will more often show atelectasis
80% of aspirated foreign bodies will be non-opaque on conventional
radiography

62. CPR in child according to American heart association in presence of


02 rescuer:
a.15 compression and 2 ventilation
b.30 compression and 2 ventilation

Answer : B

63. case about someone stabbed in the 4th right intercostal space, what
the structure behind it?
right horizontal fissure
right apical lung
answer : A
1) Horizontal Fissure: 4th Intercostal Space OR inferior border of 4th Rib
2) Oblique Fissure: 6th Intercostal Space OR inferior border of the 6th
Rib
Reference: https://quizlet.com/7143345/lungs-pleura-mediastinum-flash-
cards/

64. basal skull fracture cavernous sinus affected>>not sure, which


muscle is intact?
trapezius
Sternocliedmastoid
Answer: -
More often, oculomotor palsy occurs together with that of other ocular
motor nerves contained in the cavernous sinus in case of skull base
fracture.
Reference: http://medind.nic.in/icf/t07/i2/icft07i2p89.pdf

DR. IMRAN ASGHAR November 28, 2016


Drimran823@gmail.com
EMERGENCY MEDICINE PAPER UPDATED VERSION TILL NOVEMER 2016

65. man make RTA he was conscious, oriented, alert , but his extremity
type of shock ?
irreversable
neuro
cardio
hemorrhagic

answer: D hemorrhagic

66. Child ate a number of iron tablets presented with severe symptoms
including constipation and bloody stool nausea and vomiting and
drowsiness how would you treat him:
IV deferoxamie.
Dialysis

Answer: A Deferoxamine is the iron-chelating agent of choice.


Deferoxamine binds absorbed iron, and the iron-deferoxamine complex
is excreted in the urine. Reference: Medscape

67. Pt with Digoxin toxicity, what will order ?


lidocaine
Fb immunoglbulin
Answer: B
Decontaminate (activated charcoal) - Digoxin-specific Ab fragments
Reference: Toronto Notes

68. ptn came to ER with multiple fracture no loss of conscious with Low
bp , tachycardia , normal RR , O2 saturation = 95% ?
IV fluid
Save airway
Answer: B
All resuscitations should be performed using Advanced Trauma Life
Support (ATLS) guidelines.For the individual physician, assessment of
the polytraumatized patient is performed using a stepwise longitudi-nal
approach, in which the airway is handled first and no procedures are
initiated until the airway is se-cured. Then, breathing and circulation are
addressed

DR. IMRAN ASGHAR November 28, 2016


Drimran823@gmail.com
EMERGENCY MEDICINE PAPER UPDATED VERSION TILL NOVEMER 2016

69-Pt eat multiple drug what do (not give time)


A. Gastric leavage
B. Activite charcoal
Answer: B *in Acute situation*
70. A patient presented to ER with ingestion of multiple iron tablets.
What is your next step?
A. Induce emesis
B. Ipecac syrup
C. Gastric lavage
D. Wait and monitor
Answer: D
if asymptomatic wait 6 h then discharge

71- female with right upper abdomen pain and fever no jaundice.. what
is the
management :
Emergent surgery.
Iv fluid and antibiotics.
Discharge
Answer: B
Mild cholangitis may present with abdominal pain, jaundice, and fever.
When transporting these pa-tients to the hospital, place the patient on a
monitor and insert an intravenous (IV) line.
Provide fluid resuscitation with IV crystalloid solution (eg, 0.9% normal
saline).
Administer parenteral antibiotics empirically after blood cultures are
drawn.
Reference: Medscape

72-MTX toxicity what to give:


A- folic acid
B- folnelic acid (something like that)
C- steroid
Answer: -

73-RTA with hypovolemic shock signs , Hg low , what to give initially


A- rengar lactate
B- packed RBC
C- whole blood transfusion
Answer: A “If there is no NS in options “

DR. IMRAN ASGHAR November 28, 2016


Drimran823@gmail.com
EMERGENCY MEDICINE PAPER UPDATED VERSION TILL NOVEMER 2016

74-male paient came to ER with stap wound and hypotension what is


your next step?
fresh frozen plasm
Iv ringer lactate
Packed RBC
Answer: B
2 large-bore IV lines with crystalloid infusion
If no response to 2 L of crystalloid, infuse 2–4 units packed red blood
cells:

75 pt with ECG showing bradycardia, what should you give?


A- atropine.
B- cardioversion.
answer: A
(check the ACLS Algorithm) .

76.long case about adult come to er complaing of diarrhea , have


weekness in body and fatigue , sud-denly he fall down , BP when he
come was 120 / 80 Now it is 90 / 60 , What is the cause ?
A- Extracelular voluim loss
B- Intracellular fluid loss !?
C- Intracellular glocse loss
Answer: A
Reference: http://chemistry.elmhurst.edu/vchembook/255fluiddeficit.html

77.Why we use CT scan in trauma pt ?


a. Can detect retropretoinum hemorrhage
b. You can see the hemorrhage with contrast
Answer: A
CT scans of the abdomen and pelvis usually are performed together,
using both IV and oral contrast. Use this study to identify injuries to
abdominal and pelvic organs and to identify bleeding in the retroperito-
neum and pelvis
Reference: Medscape

DR. IMRAN ASGHAR November 28, 2016


Drimran823@gmail.com
EMERGENCY MEDICINE PAPER UPDATED VERSION TILL NOVEMER 2016

78. boy play basket ball he came with abdominal pain without any injury
in match physical exam was tenderness in paramblicus what you next :
a. Chest xray
b .abdominal CT
c. 24 recheck
d. kidney US
Answer: C
Exercise Related Transient Abdominal Pain
1. Diagnosis based
primarily on hx Transient abdominal pain during exercise - relieved
w/cessation of activity
Pain described as well localized, commonly in lumbar region of
abdomen Usually on right side
2. Physical exam: Normal abdominal exam
Abnormal abdominal exam suggests different etiology of pain during
exercise
3. Diagnostic testing :
None recommended unless physical exam or hx suggests etiology other
than ETAP
Reference:
https://mospace.umsystem.edu/xmlui/bitstream/handle/10355/13721/Ex
erciseRelated-TransientAbdominalPain.pdf?sequence=1

79.-Pt came to ER , airway pt gasping ,, u gave 2 breath by mask , pulse


is rapid & wee What to do?!
Waiting code team
Put pt in recovery position
Do CPR
intubation
Answer: C

80.Dm pt e - DKA what to give?


1 L of normal saline
Dextrose 50% followed by insulin
Answer: A
104- Comatosed patient with respiratory depression and pinpointed
pupil, drug abuse suspected what is the cause ?
Cocaine
Opiates
answer: B

DR. IMRAN ASGHAR November 28, 2016


Drimran823@gmail.com
EMERGENCY MEDICINE PAPER UPDATED VERSION TILL NOVEMER 2016

81 post RTA in ICU presnt with significant blood losse Hypotension Now
in Multi Organ failure what Is the most Organ causing other organ failure
?
heart
lung
kidney
liver
Answer:

82. Present to ER with severe headache and LP showed blood :


ruptured berry aneurysm,
epidural hematoma
Answer: A
Reference: Medscape * subarachnoid hemorrhage *

83. Pt. Overdosed a medication and presented Comatose, dilated pupil,


hyperreflexia, what is the meds:
Erdophnium
SSRI
TCA
answer: B

84. Drug addicted .. Unconsious Came with no gag reflex, What would
you do?
Intubation
Gastric lavage
give nalexone
answer: A
Airway protection by endotracheal intubation should be performed early
in the poisoned patient with depressed mental status, unless the cause
is easily reversible (eg, opioid intoxication or hypoglycemia), because of
the high risk for aspiration and its associated complications, particularly
when gastric decon-tamination procedures need to be undertaken
http://www.uptodate.com/contents/general-approach-to-drug-poisoning-
in-adults

DR. IMRAN ASGHAR November 28, 2016


Drimran823@gmail.com
EMERGENCY MEDICINE PAPER UPDATED VERSION TILL NOVEMER 2016

85-RTA pt came to ER after 30 mins o/e chest there's sound bilateral +


distended neck, Dx?!
A- Hemothorax
B- Pneumothorax
C- Cardiac tamponade
Answer: C

86-MI patient presented to ER after resuscitation he developed coma


and then died, what postmor-tem change you will find:
A- interventricular hemorrhage
B- brown coloured area supplied by middle menengieal artey
Answer: -
O n completion of the staining the infracted myocardium will show up as
pale pink colored area as against the bright red colored normal
myocardium. Even old infarcts and scars get delineated. In ab-sence of
infarction, both the surface of the slice will show uniform bright red
coloration

87-Road accedint presented with paralsis of 4 limb and extremity pink


and hot what is the type of shock?
Irreversable
Nurogenic
Hypotensive
Answer: B

88. girl come to ER complaing of RT hypochondrial pain , vomiting ,


fever with similer attacks Before On examination splenomegaly and Mild
jaundice US show gallstones...your dx?
thalacemia
SCA
answer: -
Both are right , Reference: Introduction To Pathology For The Physical
Therapist Assistant

89. drug .. Came with eye dilatation What does he takes:


TCA.
Organophosphorus .
Answer:A

DR. IMRAN ASGHAR November 28, 2016


Drimran823@gmail.com
EMERGENCY MEDICINE PAPER UPDATED VERSION TILL NOVEMER 2016

90-A 6-year-old girl, brought by parents to ER with history of falling from


height...
Not talking but crying, withdrawal from pain, open her eye only in
response to Doctor talking…,Calcu-late GCS.
9
10
11
12
Answer: B

91-Male with abdominal stab wound, vital signs ( hypotension,


tachycardia) ,What to give him first:
A. Ringer lactate
B. Fresh frozen plasma
C. Whole blood
D. Packed RBCs
Answer: A

92. 5 year old with blunt abdominal trauma, Upper GI series showed coil
spin in 2nd 3rd duodenal and high amylase, How to manage:
aparotomy and hematoma evacuation
gastroenterostomy
bowel rest
CT guided needle for hematoma extraction
Answer: C *Not sure*

93-Pts playing tennis, something bites him (birds I think) after few
minutes he has respiratory dis-tress and he was collapsed what is the
immediate tratment for this patient?
Antihistamine
Epinephrine
Answer: B
Anaphylaxis

94. Case scenario about patient who had injury in his right knee, the
doctor applied a valgus stress to examine which ligament:
A. Anterior cruciate ligament
B. Posterior cruciate ligament
C. Fibular collateral
D. Tibial collateral
Answer: D
medial collateral (tibial collateral) ligament
DR. IMRAN ASGHAR November 28, 2016
Drimran823@gmail.com
EMERGENCY MEDICINE PAPER UPDATED VERSION TILL NOVEMER 2016

95- ptn came to ER with multiple fracture no loss of conscious with Low
bp , tachycardia , normal RR , O2 saturation = 95% ?
IV fluid
Save airway
answer: B
All resuscitations should be performed using Advanced Trauma Life
Support (ATLS) guidelines.For the individual physician, assessment of
the polytraumatized patient is performed using a stepwise longitudi-nal
approach, in which the airway is handled first and no procedures are
initiated until the airway is se-cured. Then, breathing and circulation are
addressed

96. Why CT is best in blunt trauma


is best in unstable patient
Better to detect on retero-peritoneum bleeding
answer: B
CT scans of the abdomen and pelvis usually are performed together,
using both IV and oral contrast. Use this study to identify injuries to
abdominal and pelvic organs and to identify bleeding in the retroperito-
neum and pelvis
Reference: Medscape

97.patient come to ER unconscious :


check pulse
examine pupils
IV fluid infusion
clear airway
Answer: D

98. MVA with weak heart sounds and silent right side of the chest, dis-
tended neck veins, next?
Needle decompression
cardiac window
Pericardiocentesis
answer: A

DR. IMRAN ASGHAR November 28, 2016


Drimran823@gmail.com
EMERGENCY MEDICINE PAPER UPDATED VERSION TILL NOVEMER 2016

99. MVA victim presented to ER with hypotension, given crystolloid his


BP return to normal, and chest tube was inserted. what to do next?
Re-examine the chest
ABG
Answer: A

100.The fastest way to clean child stomach with iron over dose after 4
hours?

gastric lavage
serup something
enema something
other option .
Answer: - Deferoxamine is the iron-chelating agent of choice.
Deferoxamine binds absorbed iron, and the iron-deferoxamine complex
is excreted in the urine. Reference: Medscape

101- a case of heat stroke?


warm
fluid
core
cooling whole body
electrolytes replacement
Answer : D
Heatstroke is a medical emergency and continues to be one of the
leading causes of preventable death in sports.[4] Rapid reduction of the
core body temperature is the cornerstone of treatment because the
duration of hyperthermia is the primary
determinant of outcome.
Reference: medscape

102-The effectiveness of ventilation during CPR measured by:


a. Chest rise
b. Pulse oximetry
c. Pulse acceleration
Answer: A

DR. IMRAN ASGHAR November 28, 2016


Drimran823@gmail.com
EMERGENCY MEDICINE PAPER UPDATED VERSION TILL NOVEMER 2016

103-Trauma Or depressed skull fracture what immediate thing to do


(ER) Intubate O2 supplement* IV fluid Answer: A
Management
- ABCs
- ensure oxygen delivery to brain through intubation and prevent
hypercarbia
- maintain BP (sBP >90)
- treat other injuries
• early neurosurgical consultation for acute and subsequent patient
management
Reference: Toronto notes.

104 A lady brought to you after she ingested high dose of


paracetamol tablets 8 hours back, Rx:
a. N-acytylcystine

105.. Baby brought to you after he ingested drug tablets from his
relative's house, initial management:
a. gastric lavage
b. charcoal

106. A child swallowed his relative's medication. What is the best


way of gastric decontamination?
a. Gastric lavage
b. Total bowel irrigation (whole bowel wash)
c. Syrup ipecac
d. Activated charcoal

107 A child was brought by his mother due to bleeding per nose; by
examination you found many bruises in his body, over his back,
abdomen and thigh, what is your diagnosis:
a. Child abuse

108. A patient comes with metabolic acidosis, an overdose with


which of the following drugs will cause such an abnormality?
a. Salicylate
Other drugs include: ethanol, isoniazid, iron, metformin, and
acetazolamide.
DR. IMRAN ASGHAR November 28, 2016
Drimran823@gmail.com
EMERGENCY MEDICINE PAPER UPDATED VERSION TILL NOVEMER 2016

109. Burn involved 3 layers of the skin called:


a. Partial thickness
b. Full thickness
c. Superficial
d. Deep
110. Cherry red skin found in:
a. Polycythema
b. CO poisoning

111. Most serious symptom of CO poisoning is:


a. Hypotension
b. Arrhythmia??
c. Cyanosis
d. Seizure

112. A patient presented to the ER with diarrhea, nausea, vomiting,


salivation, lacrimation and abdominal cramps. What do you
suspect?
a. Organophosphate poisoning

113. Patient developed lightheadedness and SOB after bee sting.


You should treat him with the following:
a. Epinephrine injection, antihistamine and IV fluid
b. Antihistamine alone

114. Patient present with high blood pressure (systolic 200),


tachycardia, mydriasis, and sweating. What is the toxicity?
a. Anticholinergic
b. Sympathomimetic
c. Tricyclic antidepressant
d. Organophosphorous compounds
(a) causes dry skin, (c) causes hypotension, and (d) causes miosis.

DR. IMRAN ASGHAR November 28, 2016


Drimran823@gmail.com
EMERGENCY MEDICINE PAPER UPDATED VERSION TILL NOVEMER 2016

115. Female after sexual attack on exam hymen tear in


a. a-2 o'clock
b. b-4
c. c-6
d. d-8
Most likely answers, I am not sure 100 %.

116. Using gastric lavage


a. Useless after 8 hours of ASA ingestion
b. No benefit after 6 hours of TCA ingestion
c. Patient should be in the right lateral position

117. Massive overdose of aspirin 50 tabs 6 hours before, asking for


the best management:
a. Urine alkalization and dialysis

118.. What is the metabolic disturbance seen with aspirin toxicity?


a. Respiratory alkalosis with metabolic acidosis

119 Opioids antidote:


a. Naloxone

120. In battered women which is true:


a. Mostly they come from poor socioeconomic area
b. Usually they marry a second violent man
c. Mostly they come to the E/R c/o from other symptoms?
d. Mostly they think that the husband responds like this because
they still have strong feeling for them

121. After accident patient with tachycardia, hypotension, what


will be your initial step:
a. Rapid IVF crystalloid
b. CT

DR. IMRAN ASGHAR November 28, 2016


Drimran823@gmail.com
EMERGENCY MEDICINE PAPER UPDATED VERSION TILL NOVEMER 2016

122. A patient with mushroom toxicity will present with


a. Constipation
b. Hallucination
c. Anhydrosis

123. Child ate overdose of iron, best immediate management


a. Gastric lavage
b. Induce vomiting manually
c. Emetic drugs
d. Ipecac
e. Activated charcoal

124. . An alcoholic patient complains of headache, dilated pupil


hyperactivity, agitation. He had history of alcohol withdrawal
last weak so ttt is
a. a-diazepam
b. b-naxtrol
c. c-haloperidol
d. d?????????

125. A child came to ER with fever, stridor, x-ray showed swollen


epiglottis, in addition to oxygen, what u will do?
a. Throat examination.
b. An emergency tracheostomy.
c. Endotracheal intubation.
d. Nasopharyngeal intubation.

126. Arterial injury is characterized by :


a. Dark in color and steady .
b. Dark in color and spurting .
c. Bright red and steady .
d. Bright red and spurting .

127. The most common cause of death on site in a burn patient is?
a. Inhalational injury.

DR. IMRAN ASGHAR November 28, 2016


Drimran823@gmail.com
EMERGENCY MEDICINE PAPER UPDATED VERSION TILL NOVEMER 2016

128. A burn patient is treated with Silver Sulfadiazine, the toxicity


of this drug can cause:
a. Leukocytosis
b. Neutropenia
c. Electrolyte disturbance
d. Hypokalemia

129.. Charcoal doesn't bind to the following toxins except:


a. CN
b. ETOH
c. Lithium
d. Cocaine
e. Chloral hydrate
It doesn’t bind to CN, ETOH, or Lithium. However, it binds to both
cocaine and chloral hydrate.

130.. All the followings are expected with IV NAC except:


a. Anaphylactoid reaction
b. Hyponatremia
c. Higher portal vein concentration than PO NAC
d. Fetal toxicity
e. Hyperglycemia

131. In corrosive injury, all are true except:


a. Acids cause coagulant necrosis
b. Alkali cause liquefactive necrosis
c. Acids don’t penetrate deeply
d. Hydrofluoric acid causes coagulative necrosis
e. Alkali injury is more serious

132. All are criteria for a toxin to be dialyzable except:


a. Low VD
b. Low protein binding
c. Low molecular weight
d. Low endogenous clearance
e. Low H2O solubility

DR. IMRAN ASGHAR November 28, 2016


Drimran823@gmail.com
EMERGENCY MEDICINE PAPER UPDATED VERSION TILL NOVEMER 2016

133. All are dialyzable toxins except:


a. Methanol
b. Lithium
c. ASA
d. ETOH
e. Amitriptyline

134. Regarding button battery; all are true except:


a. High risk for lead/ mercury toxicity
b. Can lead to nasal septal perforation
c. Can be treated conservatively if passed Gastroesophageal
junction
d. Endoscopy should be done A.S.A.P if lodged in the esophagus.

135. All the followings indicate poisonous snakes except:


a. Heat-Sensitive pits
b. Red on yellow strips
c. Anterior fangs
d. Elliptical pupil
e. Triangular head
All of the characters mentioned indicate poisonous snakes (red-black
strips indicate non-poisonous).
136. Saline diuresis increases clearance of all these toxins except:
a. Lithium X
b. ASA
c. Iodide
d. Meprobamate X
e. Cyclophosphamide

137. Alkaline diuresis increases clearance of all the following


toxins except:
a. ASA
b. Fluoride
c. Phenobarbital
d. TCA
e. Chlorpropamide

DR. IMRAN ASGHAR November 28, 2016


Drimran823@gmail.com
EMERGENCY MEDICINE PAPER UPDATED VERSION TILL NOVEMER 2016

138. All are hepatotoxins except:


a. ETOH
b. CCL4
c. Jimson weed
d. APAP
e. Amanita phalloides
?? All are considered toxic to the liver

139. Regarding use of Atropine in Organophosphate OD, all are


true except:
a. The goal is to restore muscle activity
b. Binds to muscarinic receptors
c. Can cause CNS agitation
d. The end point is to dry all secretions
e. No maximum dose

140. All the following are indications for IV NAC in chronic APAP
OD except:
a. APAP Level › 10
b. › 7.5g in 24h in adult
c. › 100 mg/kg in 24h in healthy kids
d. APAP Level ‹ 10 + normal AST + RUQ pain/vomiting
e. APAP Level ‹ 10 + AST X2
It should be > 150 mg/kg in children.

141.. All could be life -threatening envenomations except:


a. Bees
b. Fire ants
c. Scorpions
d. Brown recluse spider
e. Black widow spider

142 .All are accepted mechanisms of CO toxicity except:


a. Cytochrome oxidase inhibition
b. Lipid peroxidation
c. Binding to cardiac myoglobin
d. Uncoupler
e. Binding to skeletal myoglobin

DR. IMRAN ASGHAR November 28, 2016


Drimran823@gmail.com
EMERGENCY MEDICINE PAPER UPDATED VERSION TILL NOVEMER 2016

143. What OD mimics Organophosphate OD:


a. Theophylline
b. Caffeine
c. Nicotine
d. Cocaine
e. TCA

144. Human bite to the hand greatest risk of infection in which


position?
a. dependent
b. clenched
c. finger extended
145. Cat bites
Mostly occur in the upper limb, and usually result in puncture wound,
thus they are very difficult to evaluate and result in higher rate of
infection than dog bites. Cat scratch disease (by Bartonella henselae) is
a possible complication of cat bites.

146. 30 year old psychiatric patient presented to ER after 5 hours


of ingestion of two safety pins, X-Ray shown it in small bowel,
What I your action:
a. Admit for surgery
b. Discharge if he is stable
c. Admit for repetitive X-Ray and abdominal exam
d. Give him tetanus toxoid

147. Which organ is affected in ingestion of overdose of


acetaminophen?
a. Liver
b. Kidney
c. Intestine
d. Stomach

148. Long scenario for a pt came to ER after RTA, splenic rupture


was clear, accurate sentence describe long term management:
Pneumococcal and meningococcal vaccines are required for capsulated
organisms.

DR. IMRAN ASGHAR November 28, 2016


Drimran823@gmail.com
EMERGENCY MEDICINE PAPER UPDATED VERSION TILL NOVEMER 2016

149. A child swallowed a battery that is shown to be in the


esophagus, what is next step?
a. a-observe for 12 hrs
b. b-surgical removal
c. c-use foley catheter to remove
d. d- remove by endoscope

150. Young aged male presented to ER after blunt trauma to


Abdomen, CT scan shows intramural hematoma, your
management is?
a. Lapratomy with evacuation of the hematoma
b. Dissection of duodenum
c. Observation

151. The CPR for child is


a. 30 chest compression-2 ventilation (1-rescuer)
b. 15 chest compressions-2 ventilation (2-rescuers)
c. 15 chest compression 1 ventilation

152. Child over-consumed a prescribed nutritional supplement


and developed abdominal pain, black vomiting, and diarrhea.
What is it?
a. Iron
b. Multivitamins

153. A child came to the ER after ingestion of multiple iron tablet


of his relative & iron concentration in his blood is 700ml what is
the best intervention
a. Gastric lavage
b. Charcoal oil
c. IV deferoxamine

154.. Patient complaining of torso pain after using tan bed, on


examination skin on the chest was red, reblenchable and
painful:
a. 1st degree burn
b. 2nd degree burn
c. 3rd degree burn
DR. IMRAN ASGHAR November 28, 2016
Drimran823@gmail.com
EMERGENCY MEDICINE PAPER UPDATED VERSION TILL NOVEMER 2016

155.. Which of the following is contraindication for nasogastric


lavage:
a. quinine
b. erosive material

156. A patient with mixed 1st & 2nd degree burns in head & neck
region, what is the most appropriate management?
a. Apply silver sulfadiazine and cream to all burned areas, cover
them and admit to hospital
b. Apply cream to 2nd degree burns and cover them, give IV fluids
c. Debridement of 2nd degree burns and …
d. Apply silver sulfadiazine then Vaseline ointment to all areas then
discharge the patient
As long as the face is involved the patient should be admitted.

157. Patient with lacrimation, salivation, diarrhea, what is the


antidote:
a. Atropine
55. Organophosphorus poisoning, what is the antidote?
a. Atropine
b. Physostigmine
c. Neostigmine
d. Pilocarpine
e. Endrophonium

158. Besides IV fluids, what is the most important drug to be given


in anaphylaxis?
a. Epinephrine
b. Steroids
c. ??? Other choices

159 About head & neck injury, which is true?


a. Hoarsness of voice & Stridor can occur with midfacial injury
b. Upper airway injury commonly occurs with midfacial injury
c. Tracheostomy is contraindicated

DR. IMRAN ASGHAR November 28, 2016


Drimran823@gmail.com
EMERGENCY MEDICINE PAPER UPDATED VERSION TILL NOVEMER 2016

160. A patient presented to the ER after a cat bite with greenish


discharge which organism:
a. staph aureus
b. pseudomonas aurgenosa
c. bacteriodes
d. strept. Viridans

161. pt come in emergency with complaint of HCL burn on her,


the skin of the pt is burnt, now emergency treatment is
a. NAHCO3
b. DEBRIDGEMENT
c. WATER IRRIGATION
d. ???

162. pt come in emergency with frozen foot, FIRST AID treatment


is
a. HEAT AND WARM AIR
b. IN WARM WATER
c. GIVE COFFE AND TEA
d. RUBBING THE FOOT

163. A child took an unknown medicine and presented in the


emergency with decreased level of consciousness, pinpoint
pupil, urination, diarrhea, diaphoresis, lacrimation, excitation,
and salivation. The treatment is
a. gastric lavage
b. activated charcoal
c. atropine
d. naloxone

164.. A young fireman come to ER complain of headache and


dizziness after some activity (they mention something I couldn’t
remember) ABG show, normal partial pressure of oxygen what is
the first step in this patient?
a. O2 therapy
b. C-xray
c. Caboxyhemoglobin level
d. Anemia evaluation
Query CO poisoning CO levels must be checked to guide our
management.
DR. IMRAN ASGHAR November 28, 2016
Drimran823@gmail.com
EMERGENCY MEDICINE PAPER UPDATED VERSION TILL NOVEMER 2016

165. Which role used to calculate burn surface area in case of


burn:
a. Nine
b. Seven

166. pt came to ER decreased level of consciousness and pinpoint


pupil?
a. opiate over dose
167. A baby fell down from stairs and came with multiple
contusions some of them were old and X-ray show fracture in
radius how to manage?
a. Splinter for his hand
b. Hospitalization and call social worker

DR. IMRAN ASGHAR November 28, 2016


Drimran823@gmail.com
My exam on 20 August 2015 Dr.Abdallah Mahmoud

1-A case of helicobacter pylori you also screen for:


A bleeding.
B Gastric outlet obstruction.
C gastric carcinoma.

2- A case of rosacea treated with multiple regimen .now painful


rash (picture)
as petechi around the eye...
What will you add:
A Topical sulfanide corticosteroid?
B Acyclovir.
C oral steroid.
D facemask of green material. I do not remember.

3-Long scenario of a case of female with excessive exercise


...complain of light intermenstural bleeding...TSH and pituitary are
normal ...what is the cause:
A defect progesterone.
B inadequate estrogen.
C ovarian failure.
D I do not remember.

4-Acase of nulliparous came e amenorrhea complaining of


polyuria, fatigue some un-related symptoms...no fever ...and lab
+ve GnRH...
What is the cause of symptoms:
GnRh
Progesterone?
Estrogen
can not remember.
5-Mechanism of action of glipizide:
Incr. Insulin availability.
Increase insulin receptor sensitivity.
Dec. Insulin resistance.
Incr. Insulin prod.
6-Mechanism of action of anti-depressant drugs:
Incr. Availability of substance.
Increase uptake by cells.

‫بالتوفيق للجميع‬ ‫وتحقيق اآلمال‬... ‫نسألكم الدعاء بصالح الحال‬


My exam on 20 August 2015 Dr.Abdallah Mahmoud

Dec. Renal excretion.


Decrease ...not remember.

7-In epedimolology:
Screen for DVT & OCP...
2 groups
1st one DVT+OCP 50 from 1000
2nd DVT AND NO OCP 150 from 9000
asking about prevalence rate in 1000?
8-In epidemiology:
Asking about 2012 cumulative incidence rate
in population of brucellosis
200 new cases.
100 old cases.
1200 dead from disease.
In addition, risk rate in mid-year was 3% percentage in 800,000
population.
What will be incidence cumulative rate in 10,000?
I hate epidemiology.

9-There is a question about Adolescent assessment called


HEADSS...
asking about what better for patient discussion:
Discuss in presence of parents.
Discuss in private.
Discuss by doctor opposite gender.
Fourth, I don't remember.
10-Acase of mother her son died 1 month ago ...she is still crying,
loss of appetite, loss of normal activities...diagnosis:
Depression.
Psychosis.
Normal bereavement.
11- A case of women fear of parks and sports events get worse to
even not going anywhere...diagnosis:
Social phobia.
Agoraphobia.
Depression.

‫بالتوفيق للجميع‬ ‫وتحقيق اآلمال‬... ‫نسألكم الدعاء بصالح الحال‬


My exam on 20 August 2015 Dr.Abdallah Mahmoud

12-Long scenario about patient with severe muscle pain and


weakness...emergency room done a lot of lab mentioned in
question
but most significant one is sickle cells in smear...
What will you do now?
O2
Narcotics.
I.V fluids.
Anti -biotic.
13- A case with low back pain...
Pet scan show early osteoporosis....
What will you do:
Add ca.
Give calcitonin?
Give estrogen.
Ca ,vit D& biphosphonates.

14-A case of midline mass below hyoid bone moves with tongue:
Thyroid enlargement.
Cystic hygroma.
Thyroglossal cyst.
Congenital lymphoma...not certain.

15-A case of conjunctivitis in 1 eye and uveitis in the other.


What topical substance you give:
Topical steroid.
Topical sulfacetamide.
Tiaramide.
Cyclophenolate(cicology)... may be
16-A Case of sudden loss of vision...not mentioned painful or
painless.
Fundus show:
Multiple hgs and edema. In addition, other finding
Cause:
Artery occlusion.
Vein occlusion...
2 other options can not remember

‫بالتوفيق للجميع‬ ‫وتحقيق اآلمال‬... ‫نسألكم الدعاء بصالح الحال‬


My exam on 20 August 2015 Dr.Abdallah Mahmoud

17-Acase of loss of vision in a patient with high glaucoma...optic


disc problem...
Asking about mechanism of this:
Aqueous humor pathology with vessels
Changes...
What is this...I do not know.

18-Atypical case as in UQU...


Apt ...his father was diagnosed with pancreatic cancer and
died...he did all labs it was normal ...He visited three doctors and
not convinced with normal results.
Diagnosis:
Conversion disorder.
Somatization.
Hypochondriasis.
Delusion.
19-Along scenario about pt. In day, care presented with arthralgia,
conjunctivitis,
and fatigue and temp 38 c....
Treatment...
Antibiotic for 3 days then NSAID.
Single dose of corticosteroids.
Antiviral drugs if within 72 from start.
Nothing.
20-Acase taking warfarin what to avoide:
Carrots.
Meat and poultry.
Green vegetables.
Not remembered.
What food to avoid?
21-Acase of sudden abdominal pain...examination show diffuse
abdominal pain and tenderness...past history of heartburn...
Diagnosis:
Acute intestinal obstruction
Perforated duodenal ulcer
Acute pancreatitis.
Myocardial infarction

‫بالتوفيق للجميع‬ ‫وتحقيق اآلمال‬... ‫نسألكم الدعاء بصالح الحال‬


My exam on 20 August 2015 Dr.Abdallah Mahmoud

22-A case of upper abdominal pain for a period of time...not


related to meals....
Investigation shows C12 urease positive....
What is the causative organism:
H.Pylori.
X.jejini.
and two others.
23-A case of normal patient but on lab was spherocytosis.
What is the most appropriate test to confirm diagnosis?
Osmotic fragility test.
Hb Electrophoresis.
24-A case of in hospital take 10 day of antibiotics.
Before discharge, the following results as follow:
All labs are within normal levels and picture of blood:
What will you give him now for long-term therapy:
Penicillin
Hydroxyurea.
Steroids.
Iron therapy.

‫بالتوفيق للجميع‬ ‫وتحقيق اآلمال‬... ‫نسألكم الدعاء بصالح الحال‬


My exam on 20 August 2015 Dr.Abdallah Mahmoud

25-I had the rt picture in my exam


Hundreds of small eiosinophilic cells(red )
With only 2 large basophilic cells (blue)
Describing a case of generalized
lymphadenopathy,splenomegaly&fatigue.
May be some lab results increased white blood cells and
differential lymphocytosis 71%
Diagnosis:
Hodgkin's lymphoma.
Non-Hodgkin's lymphoma.
Leukemia...may be
other choices I do not remember.
I do not know the answer.
Finally, I passed ...alhmdukilah.

‫بالتوفيق للجميع‬ ‫وتحقيق اآلمال‬... ‫نسألكم الدعاء بصالح الحال‬


My exam on 20 August 2015 Dr.Abdallah Mahmoud

26-Acase of generalized muscle pain cough and


conjunctivitis...she developed red
Maculopapular rash all over the body...on examination ill patient
with low grade fever multiple maculopapular rash all over his body
.clear vesicles shown in back. Normal muscle power and normal
sensation....
Most likely diagnosis is:
Chicken pox.
Shingles.
Measles.
Gulian bare syndrome.
27-A fatty child come with his mother for evaluation as he eating
more ...
He is in 97 percentile...
What will you do:
Screen for hyperlipidemia?
Life style modification.
Two other options.
28-Apatient come for routine check-up...
His BMI is 28 and no complain of any kind.
He just concerned about his blood pressure.
When you checked him pressure was 135/89

‫بالتوفيق للجميع‬ ‫وتحقيق اآلمال‬... ‫نسألكم الدعاء بصالح الحال‬


My exam on 20 August 2015 Dr.Abdallah Mahmoud

And 137/90 in 2 repitive readings with 5 minutes apart.


He asked you how he could reduce his pressure. What is the most
appropriate advice to him:
Reduce salt in your food.
Body weight reduction and exercise.
Eating healthy food.
Eating fewer sugars.
29-Acase of infectious mononucleosis
Treatment:
Acyclovir and i.v antibiotics.
Steroids.
Admission.
Paracetamol and rest.
30-A case of fatigue, dysnea,spoon shaped and brittle nails.
Hb 10
MCV 69
Retics 1.2
TIBC 72 normal up to 42.
Diagnosis;
Perncious anemia.
Iron def. Anemia.
B12 def. Anemia.
Thalassemia trait.
31-A case of pulmonary edema (not typical scenario)
Best drug for treatment:
Hydrochlorothiazide.
Nitrates.
Digoxin.
Furosemide.
32-A case of soft enlarged prostate with urinary symptoms.
Best drugs is
Indapemide.
Titracin
Prazocin.
Acetazolamide.

33-A case of man in raw waiting for his turn for long time.
Suddenly he fall down.
He wake again fast with no residual effects. He witnessed before
syncope yawning. Normal vital signs.

‫بالتوفيق للجميع‬ ‫وتحقيق اآلمال‬... ‫نسألكم الدعاء بصالح الحال‬


My exam on 20 August 2015 Dr.Abdallah Mahmoud

Diagnosis:
Fainting.
Transient ischemic attack.
Unusual fatigue never had before.
Other choice I can't remember.

34-Which lipid to screen for ischemic heart disease:


Cholesterol.
Triglycerides.

35-Female with urinary incontinence with cough and laugh...no


other symptoms.
On examination normal pelvic structure except for mild cystocele.
Treatment:
Surgery.
Kegel exercise.
Alpha-blockers.

36-Most common predisposing factor of UTI in females:


Whipping from back to front.
Wearing wet tampons.
Tampons for a very long time.
Other i dont remember.

37-A case of child diagnosed with malaria (falciparum)


severe toxic pale patient with high fever.
Treatment with chloroquine.
Dose:
600mg 1st then 600 after 6 hours.
600 then 300."
300 then 600."
300 then 300"

38-Diabetic patient with eye problems.


Neuropathy and fungal infection.
On lab, you found
RBS 300 mg/dl.
Hb A1C 13%.
Proteinuria.
He is on metformin 500 mg 2 times per day.
What will you do:

‫بالتوفيق للجميع‬ ‫وتحقيق اآلمال‬... ‫نسألكم الدعاء بصالح الحال‬


My exam on 20 August 2015 Dr.Abdallah Mahmoud

Metformin 1gm 2 per day?


Add 1 dose of long acting insulin.
Switch to two doses of mixed insulin.
Add sulfonylurea drug.

39-What is the best diagnostic test for diabetic nephropathy:


Urine deistic.
Kidney function test.
Albumin/creatinine spot ratio.
24h urinary proteins.

40-Pt 34 years old came for routine follow up. Examination is


normal but he is concerned about his dad who died in age of forty
by heart attack. You ordered lab which is as the following^
RBS 160 mg/dl.
Hb A1C 5.8%
Cholesterol 179 normal up to 200.
LDL 90 normal below 100.
What should be most significant next year follow up
Cholesterol?
Blood sugar.
LDL
PSA level.

41-NICU child suddenly developed distress with absent breath


sound.
You will put Butterfly needle in
second intercostal space.
3rd.
4th.
Fifth space.

42-Patient with mild urinary symptoms .on examination slightly


enlarged soft prostate.
U/S show middle lobe enlargement's normal
what this patient need:
Periodic PSA level.
Cystoscopy.
Beta-blockers.
Do nothing.

‫بالتوفيق للجميع‬ ‫وتحقيق اآلمال‬... ‫نسألكم الدعاء بصالح الحال‬


My exam on 20 August 2015 Dr.Abdallah Mahmoud

43-Pt came to emergency room after accident.


He is in distress; diaphoric but conscious.
Vitals as follows:
Bl/p 100/70
Pulse 110
R.R 40
You found injured chest e scattered trauma sites around lt upper
limb. You ordered x-ray
and came as the following...
What will you do:
Thoracocntesis?
Chest tube insertion.
Pericardiocentesis.
Thoracotomy.

44-Acase of pt. with severe stridor at night.


He was (word not known to me) during conversation.
On examination, you found enlarged tonsils.
What will you do:
Adenectomy?
Inferior turbinectomy.
Neck surgery.

45-A case of vaginal pain and discharge.


On wet preparation show flagella.
Diagnosis:
Trichomoniasis.
Chlamydia.
Trichinosis.
Leshmaniasis.
46, 47: There was two questions about dementia
1st measure to prevent or nothing to do.

‫بالتوفيق للجميع‬ ‫وتحقيق اآلمال‬... ‫نسألكم الدعاء بصالح الحال‬


My exam on 20 August 2015 Dr.Abdallah Mahmoud

Second about underlying pathology in dementia with


Parkinsonism.

48-There was a question about zoster ophthamicus.

49-There was a question about muscle pain, fatigue, dysphagia


and recurrent pneumonia .In examination sluggish reflexes and
spastic muscles.
Diagnosis:
Myasthenia gravis.
Motor neuron disease.
Dermatomyositis.
Other option not remembered.

50-There was a question about neuron case ...could not remember


details but I clearly remember there was brain MRI with unilateral
brain atrophy like the following picture but in exam changes only in
one side; other side is normal.
Diagnosis:
Tuberous sclerosis.
Three other options I cant remember because I choose this.

51-37 year's old chronic renal pt. came with muscle pain and
irritability.
On examination tapping in front of ear

‫بالتوفيق للجميع‬ ‫وتحقيق اآلمال‬... ‫نسألكم الدعاء بصالح الحال‬


My exam on 20 August 2015 Dr.Abdallah Mahmoud

Causing sudden jaw movement.


You ordered lab, which came as the following:
Na 138
Cl 99
K 4.2
Ca 6.7
Creatinine 1.8
PTH high level
1(OH) cholecalciferol normal level.
What is the cause of these patient symptoms:
Vit D deficiency?
Parathyroid problem.
Renal disease.
Osteoporosis.
Defect calcium intake.

52-There was a case of major depression came to clinic and when


you talked to him, he told you about detailed plan for suicide...
What are you going to do
Increase dose of medication.
Admit to hospital.
Inform police.
Discharge and follow up.

53-A case of finger pain for 2 days.


You prescribed antibiotics and NSAID.
After 2 days pain still present with picture like that...
What will you do:
Regional anesthesia and drainage?
General anesthesia and drainage.
Add i.v antibiotics.
Add anti-crystal medication.

‫بالتوفيق للجميع‬ ‫وتحقيق اآلمال‬... ‫نسألكم الدعاء بصالح الحال‬


My exam on 20 August 2015 Dr.Abdallah Mahmoud

54-A case of constipation in lt lower abdomen.


On examination there was small mass in lt lower quadrant.
No history of weight loss.
There was other details.
Diagnosis:
Crohn`s disease.
Diverticulosis.
Other two options.

55-A case of hematoma with hemorrhoids tried multiple drugs what


will you do?

56-Regarding D&C
which one is right:
Antibiotic is mandatory.
Use of Anesthesia decrease risk.
Small cupped probe is better than wide.
Broad cupped probe is better to decrease perforation.
Add blood and oxytocin prior to surgery.

57, 58: There were two cases of aortic dissection.


Do not forget radio-femoral delay it is the key.
59-30 years old female G 3 PO ...12 weeks gestation. She had a
history of two previous abortion. She done genetic counseling
which show aneuploidy XO...
What is the possibility of this aneuploidy in current conception:
40%
50%
60%
70%?

‫بالتوفيق للجميع‬ ‫وتحقيق اآلمال‬... ‫نسألكم الدعاء بصالح الحال‬


My exam on 20 August 2015 Dr.Abdallah Mahmoud

60-Female pt. G 3 p 2...14 weeks gestation came with lower


abdominal pain and bleeding. She looks anxious and pale...
On ex:
Bl/p 100/70
RR 22
TEP 37.8
PULSE 100.
U/S show gestational sac and viable fetus.
In pelvic examination:
It show closed cervix and copious bleeding.
Diagnosis:
Missed abortion.
Inevitable abortion.
Threatened abortion.
Incomplete abortion.

61-child with fever and ear pain..O/E: .plugging tympanic


membrane.
-Serous otitis media
-otitis externa
-perforation -Ramsey hunt syndrome.

62-There was a case of pregnant female 24 wks.


Gestation>>.There was history of conscious level abnormality...
No peripheral edema and +2 proteinuria.
On exam;
Good general condition; normal vital signs except bl/p
159/95...some tremors.
Fetal heart monitoring was normal...
What is the treatment for this case;
Methyldopa.
Hydrochlorothiazide.
Mg so4.
B blockers. Not sure.

‫بالتوفيق للجميع‬ ‫وتحقيق اآلمال‬... ‫نسألكم الدعاء بصالح الحال‬


child with erythema and itching and scaling in front of both elbows,  
behind knees , face ..your diagnosis? 
a­ Contact dermatitis  
b­ Scabies  
c­ Eczema 
­A young male that was previously diagnosed of having HIV , recently developed  
purple lesions in the body and oral cavity , what is the best treatment : 
­Oral antibiotics 
­Topical antibiotics 
­Steroids 
­Chemotherapy and Radiation 
 
­A man complains of a penile discharge after an un protected sex , culture showed  
gram negative diplococcic, what is the diagnosis : 
­Syphilis 
­Gonococcal urethritis 
 
Longest scenario you will be ever seen ; about 10­15 lines and each answer 2  
lines . Briefly : old man known case of DM , HTN on medication complain of syncope  
when he playing with his grandson associated with sweating . It is rapid onset and  
rapid recovery .His daughter said that her father completely normal regarding his  
mental and behavior status . Past history of medical admission couple of months  
under indication of shortness of breath / chest pain which was completely normal .  
On examination : Vital signs are stable including normal BP, Ejection systolic  
murmur over left sternal border : 
a) Decrease dose of antihyptensive to 5 mg ( it was one drugs of diuretics and  
the dose is 10 mg ) as well as DM medication 
b) Admit to hospital to cardiac series / investigation 
c) Order Immediate ECG 
d) Reassure him that this syncope due to effect of DM on autonomic nervous  
system 
Patient presented with nausea, vomiting, nystagmus, tinnitus and inability to walk unless he 
concentrates  
well on a target object. His Cerebellar function is intact, what is the diagnosis?  
a) Benign positional vertigo 
b) Meniere’s disease 
c) Vestibular neuritis 
 
Duration of drug in Rheumatoid fever is : 
a) 6 years 
b) 15 years 
c) Primary prevention lasts for 10 days and 2ry prevention lasts for 5years or 10 years 
depending on  
presence of cardiatis 
Options were  3 months 6 months 5 years 15 years  
 
 
1­in epidemic research...a test chosen as gold stander for septicemia in 200  
neonate...among 50 neonate who diagnosed with sepsis by gold standerd thes  
the test was positive in 35 neonate,,,among 150 neonate who diagnosed aseptic  
by this test ,,the test was negative in 25 neonate,,,,,what is the sensitivity of this  
test? 
­80% 
­70% 
­30% 
­90% 
6 or 38 week. After delivery she started to bleed from nose and other places.  
Cause? 
A­ DIC 
B­ Factor V Leiden 
Scenario was bit different with different options.were ITP gestational , ITP 
 
Male, with history of unprotected sex with unknown woman. Gram negative  
diplococcic. With picture of his penis showing discharge. 
A­ N. Gonorrhea 
Exact same question with pic 
 
Depressed patient has injestion big quantity of Aspirin 6 hours ago, came to ER complaining of 
nauesa,  
vomiting, increase respiration, investigatin showed highly elevated level of ASA, what is your 
action: 
a) urine acidity something 
b) charcoal 
c) haemodialysis 
d) Alkalinization of the urine 
 
scheduled for elective surgery at the morning .He is fasting from  
midnight .Which regime you will give him :  
a) Half dose at the morning  
b) Half dose at morning and half dose at the midnight 
c) Usual insulin dose 
d) Omit the scheduled surgery dose 
female with negative pap smear you should advice to repeat pap smear every: 
a) 6m 
b) 12m 
c) 18m 
d) no repeat 
Options were different 
I chose after 3 negative no need for further test but I think it is wrong 
 
­old women work in neonate nusry unit present with recurrent conjunctivitis  
the best way to prevent this problem: 
­wear gloves 
­wash hands frequrntly 
pt. on treatment for skin rash macules.papules,pustules and viscles whole the  
body…he gave history of malaise,fever and headach one day bafore the  
rash,,,which of the following viral is most likely the cause: 
­herpis simlex type 6 
­EBV 
­cytomegalovirus 
­varicella zoster. 
Shingle was written in place of VZ 
 
pt with ear trauma since 2year presented with dischange and decrease hearing  
he take several course of antibiotic without emprovement O/E..there is  
perforation in the tempanic mem. And conductive hearing loss..which is the  
appropriate mx: 
­mastoidectomy 
­topical antibiotic 
­maryngoplasty 
­systemic steroid 
No option mryngoplasty but systemic antibiotic written 
 
during rape rupture of hymen at  
a)2 
 4   
6  
8 o’clock 
Answer is 6 
 
0­pt. presented with cold intolerance ,bradycardia, depression,  
constipation…most propable diagnosis: 
­hyperthyroidism 
­hypothyroidism 
­addison disease 
 
 
DHA­10qA girl with 10th percentile for height  being examined by physician. At what time after 
menarche         spinal length stops? 
a. 24 months 
b. 36 months 
c. 8 months 
 
17 years old , she missed her second dose of varicella vaccine the first about 1 y  
ago what you'll do: 
a­ give her the second dose only 
 
­ Painful loss of vision: 
a­ Acute glaucoma 
 
Child presented with black swelling 1X1 cm in inner lower lip, not tender,  
suddenly discovered (dental problem why I should answer it): 
a. Gingival cyst, 
b. Tumors 
There were strange unreadable options  
 
7­patient developed fever followed by macules, papules and pustules in the back with  
erythema and pain at the site of lesions, what is the diagnosis: 
a­chicken pox 
b­HSV1 
c­shingles 
d­measles 
patient came with palpitation, not had any disease history, not used any cardio  
stimulatory drugs or alcohol and not had chest pain, PR was 210 otherwise normal  
examination and ECG inconclusive, what is the most appropriate management: 
a­compute P­R interval 
b­cardiac enzymes 
c­V/Q scan 
patient diagnosed as depression, what is the initial pharmacotherapy: 
a­SSRI 
b­MOAI 
c­TCA 
 
in a patient who is on antipsychotic and he is noncompliant to therapy, how the  
physician can response: 
a­give depot injection of haloperidol or fluphenazine 
b­give IV antipsycotics 
c­give clozipine and other one orally?? 
d­give two other drugs orally?? 
Sorry some choices I couldn’t remember the name of the drugs in the choice 
 
6 years old child was bitten by a CAT , what is the organism ? 
a­ Pasteurella 
b­ Strepto viridians 
c­ Staphylococcus 
Options different . Spirochet gram postve rods gram negative rods 
 
45 years male, last few weeks increase alcoholic intake, activity and phone calls, also  
no sleeping for more than 2 hours at a atime. What is your diagnosis : 
a) alcohol abuse 
b)mania 
 
60 years female, c/o back pain, bone density=2.5, what will you do : 
a)NSAID 
b)calcium 
c)calcium+vit.d+phosphorus 
d)exercise advise 
 
male presented with white discharge of urethra, febrile, dysurea , gram stain show  
gram +ve diplococcic. What is your diagnosis : 
a)gonorrhea 
 
6­pt with Hx of unprotected sex…cam with penile discharge …culture  
done and revealed gram negative diplococcic, Associated picture of the  
discharge and the gram stain Your diagnosis: 
a­chlamydia 
b­gonorrhea 
c­strept 
d­staph 
I had both these two questions about gonorrhea  
 
A patient complaining of S.O.B , on examination one nostril is edematous and  
blocked , what is the best INITIAL management : 
­Decongestatns 
­Sympathomimetics 
­Corticosteroids 
­Antihistamines 
This kind of questions but different. H influenza virus was written and in options their was 
combination. 
Only combination with anti viral I selected 
 
 
A new mother brings her 2 weeks old baby saying that he has problem in breathing  
and he is dying , on examination the baby is normal , the mother has : 
­post partum psychosis 
Senrio was different wording. But case samE 
 
Post partum bleeding from injection site 
F d p product was in options 
 
After Fever treatment suddenly drenching swet 
Answer defevrcnce 
 
Picture of distal finger with red papule, it was painful and was  
treated for one weak with Augmentin with no cure: 
surgical excision and drainage under general anesthesia 
surgical excision and drainage under regional anesthesia 
continue augmentin for another weak 
change antibiotic 
 
­in epidemic research...a test chosen as gold stander for septicemia in 200  
neonate...among 50 neonate who diagnosed with sepsis by gold standerd thes  
the test was positive in 35 neonate,,,among 150 neonate who diagnosed aseptic  
by this test ,,the test was negative in 25 neonate,,,,,what is the sensitivity of this  
test? 
­80% 
­70% 
­30% 
­90% 
1. Child with aspirin intake overdose ...what kind of acid base balance: 
a) Metabolic alkalosis wt respiratory 
b) Metabolic acidosis wt respiratory alkalosis 
c) Respiratory alkalosis with metabolic acidosid 
d) Respiratory acidosis with metabolic alkalosis 
 
Patient has sudden Rt eye pain ,red with dilated pupil , cloudy cornea and  
increased IOP , left eye by examination has cupping disc , and normal IOP,  
diagnosis 
a­ Rt glaucoma , left glaucoma 
b­ Rt uvitis and Lf retinal degeneration disease 
c­Rt conjunctivitis and left reflex symptoms 
Wording was different . Option was glucoma most appropriate 
 
20 yrs old man NOT KNOWN TO HAVE MEDICAL PROBLEM PRESENT C/O  
increase heart beat ( PALPTATION ) , NO CHEST PAIN , NO DYSPNEA OR  
COUGH , OE: ALL NORAML , CXR: ­VE , BP 135 /110 , ECG >> 210 BPM >> NO  
INJURY EVIDENCE . WHAT THE NEXT STEP >> 
A­ COMPUTED P­R INTERVAL 
B­ V/Q SCAN 
C­ CARDIAC ENZYME 
 
12/Paranoid personality disorder: 
A­most prevelent personality disorder 
B­lead to paranoid shizophrenia 
C­needs high antipsychotic medication to manage 
D­somthing about mistrust 
 
Megaloblast anemia 
 
Itp splenmegaly. Pregnant . Gum bleeding 
 
40/AB of case of meningitis>>penicillin 
 
1­pt with DM and obese ,plane to reduce his wt is : 
a.decrease calori intake in day time  
b.decrease calori and increase fat  
c.decrease by 500 kcal/kg per week 
d.decrease 800 per day 
 
child with erythema and itching and scaling in front of both elbows,  
behind knees , face ..your diagnosis? 
a­ Contact dermatitis  
b­ Scabies  
c­ Eczema 
 
Infantinle colic what will you tell parents 
Options 
Reassure 
Antispasmodic 
Umulqurah: 
 Depressed patient has injestion big quantity of Aspirin 6 hours ago, came to ER complaining of 
nauesa,  
vomiting, increase respiration, investigatin showed highly elevated level of ASA, what is your 
action: 
a) urine acidity something 
b) charcoal 
c) haemodialysis 
d) Alkalinization of the urine 
 
patient with DM and obese ,plane to reduce his wt is : 
a) Decrease calorie intake in day time  
b) Decrease calorie and increase fat  
c) Decrease by 500 kcal/kg per week 
d) Decrease 800 per day 
 
yersinia pestis 
Cattle 
Dog sliva 
Cat 
 
Old patient diagnosis berret esophagus what you advise 
Periodic endoscopy biopsy 
Laparoscopic Fundoplication 
 
Post coital bleed . Tumor on labia majora what will you do 
Pap smear 
Biopsy 
 
Senerio polycystic kidney options 
Familial hypopalsttic something 
Atherosclerosis kidney 
Strange options can't remember 
 
 splenmegaly. Pregnant . Gum bleeding. Low platelet 
Gestational thrombocytopenia 
ITP 
 
Testosterone injection to HIV  
Lean body mass 
Oppose estrogen  
Antifungal 
 
Depression immediately treatment 
Diazepam 
Ssri 
 
Long case dementia what you advise 
Hyoscimine decrease rate of memmory loss 
Exercise  
Daily routine activities to b changed now 
 
Post partum bleeding Rx 
Uterine relaxant 
Uterine contractor 
 
Asthma in pregnancy contra indication 
Prostaglandin 
Methgyne forget exactly 
 
School girl depression .pluck own hairs treatment 
Ssri 
Diazepam 
Mao 
 
Silver wiring  
Psoriasis 
 
Patient smoke inhalation drug 2 hours he lost association time and space. 
Canabis 
Nicotine 
Caffeine 
Cocain 
 
Diabetic nephropaty screening test 
Options  
Urine dip stik 
Ultrasound kidney 
Urine sedimen 
urea creatinine. 
 
female who does a lot of training and had amenorrhea for 5 months, this can  
increased her risk of: 
a­ovarian cancer 
b­endometrial cancer 
c­osteoporosis 
d­infertility 
With different worindin Nd options. 
Ans osteoporosis 
1.Lactating women with mastitis:

a) Continue lactation

b) Clean with alcohol

c) Surgical drainage

2.The management of breast engorgement:

a) Cold compression with stoppage of breast feeding

b) Cloxacillin with continue breast feeding

c) Warm compression with continue breast feeding

3.Pregnant female developed gestational diabetes that was not controlled by


diet and was switched to insulin. She is at great risk later in life to develop:

a) Diabetes type 1

b) Diabetes type 2

c) Hypoglycemic attacks

4.A female with foul odor green vaginal discharge with flaglated under the
microscope, Dx:

a) Trichomonas
b) Bacterial viginosis
c) Syphilis
d) Lichen planus
5.Pregnant lady 16 w GA on U/S fetus small for age, P/E uterus size 12w, what
is the diagnosis:

a) Chorionic carcinoma

b) Hydatiform mole

c) Tumor at placenta

d) Toxemia Of Pregnancy

6.Female patient wear glass since 10 years , she diagnosed recently type 2 DM
, she should screen or examine her eyes every:

a) 6 months

b) 12 months

c) 2 years

d) 5 years

7.One case of Hand foot mouth disease AND ASKED CAUSATIVE AGENT

A) Coxsackievirus
B) rota virus
C) CMV

8.Young female with whistish grey vaginal discharge KOH test.smell fish
like, with clue cells on microscopy.What is the diagnosis?
A) Gonorrhea
B) candida
C) traichomanous vaginalis
D) bacterial vaginosis
9. Bad breath smell with seek like structure, no dental caries & Ix are
normal, what's the likely cause:

a) Cryptic tonsillitis
b) Sjogren Syndrome
c) GERD

10. One question about DM patient on short acting and long acting
insulin regimes.One graph was shown,which demonstrated High sugar
levels at night and early morning.Which insulin dosage should be
increased.
A)increase evening short acting Insulin
b)increase evening long acting Insulin

11. 5 years old. Enuresis. What will you tell the parents?
A- Reassure
B- Use star chart
C- Use star chart + Moisture Alarm
D- Use star chart + Moisture Alarm+ Desmopressin

12. Person presented with severe occipital headache.Ct scan picture was
given
13. A scenario about a pt with HIV who now has symptoms consistent
with TB.sputum culture was positive but Mantoux test was
negative.what is the explanation?

a) Mantoux is not the test for screening TB


b) This is a false negative result because there is a defect in the delayed
immunity of this pt
c) It should b confirmed by heaf test

14. Town with population of 1500 , total births 83,stillbirth 3,total live
birth 80
Calculate stillbirth rate
a- 36.1
b- 75.3
c- 43
d-37.5

15. A scenario about a middle aged man having features of


hypocalcemia(10.4mg/dl), hyperphosphatemia(3.4mg/dl) and PTH level
around 401…..Off and on having renal colics, US showing multiple
renal calculi, he also gave h/o multiple fractures. Labs show low
calcium, markedly elevated PTH,wt u will do?
A. Observe
B. Bisphosphonates
C. Ca and vit D
D. Start workup for parathyroid adenoma

16. A child developed pain and discharge from his ear, on examination
there was a discharge from the ear canal and sever pain upon pulling
the pinna of the ear out, the diagnosis is :
A. -Otitis media
B. -Otitis externa
17. Child presented with decreased hearing for 1 year, on exam. there is
fluid behind the ear drum and
adenoid hypertrophy. In addition to adenoidectomy what will you do:
a) Myringotomy.
b) Gromet tube insertion.
c) Antibiotics.

18. Patient with Hodgkin’s lymphoma and red strunberg cell in


pathology and there is eosinophil Lymphocyte,histiocytes in blood and
absence of fibrous bands .so pathological classification is:
a) Mixed-cellularity subtype
b) Nodular sclerosis subtype of Hodgkin's lymphoma

19.Baby born to mother vaginally develop fever then vesicular rash


start at face
and axial distribution then all over body , the best treatment
A- Acyclovir
B- Varicella zoster immunoglobulin
C- Antibiotics
D- Steroids

20.Patient had history of pancreatic cancer on chemotherapy then


improved completely came to doctor concerning about symptoms of
recurrence and mentions a history of visiting three hospitals.But once
hospitals were contacted there was no record found.Once reality
discussed with patient he leaves hospital premises immediately. This
patient has:
a) Malingering
b) Hypochondriasis
c) Factitious Disorder
d) Conversion Disorder
21. Which subtype of bipolar disorder type 2 best responsive to lithium
dysphoric
rapid cycling
mixed
classic mania

22. 10 years old child with rheumatic fever treated early, no cardiac
complication. Best to advice the family to continue prophylaxis for:
a) 5 months
b) 3 years
c) 6 years
d) 15 years

23.Patient with hx of acute otitis media , came with cloudy discharge


from his left ear you should manage him by :
a) topical antibiotic
b) systemic antibiotic
c) steroids

24. 37 years old post cholecystectomy came with unilateral face swelling
and tenderness. Past history of mumps when he was young. On
examination moist mouth, slightly cloudy saliva with neutrophil and
band cells. Culture of saliva wasn't diagnostic. What is the diagnosis?
a) Sjogren Syndrome
b) Parotid cancer
c) Bacterial Sialadenitis

25. Patient presents with vomiting diarrhea, abdominal pain also and pale
skin and brown nails and with irritable leg and more ----dx
-irritable/Restless leg syndrome
-clostridium difficile
-Chronic renal failure
26. Pt complain of central lower back pain when he wake up at the morning,
stay for 30 min without medication, with slight improvement brought by
NSAID, investigation shows lumber spinal stenosis, and O/E we just found
Para spinal muscle spasm, otherwise normal, what will you do:
a- Physical therapy
b- Steroid injection
c- Surgery

27.A case of Obstructive sleep apnea with other co morbid with pacemaker
installed for heart functioningPatient had already gain ideal weight and being
treated with CPAP but no response.whats next:
a.nasal surgery
b.pharyngyeal surgery

c.atrial pacing

d.Tx with acetazolamide & o2

28. One case Of unilateral gynaecomastia in puberty.What will be the


treatment:

Re assurance

Estrogens

29.One scenario of diagnosed Cushing syndrome.She will be at risk of

Osteopenia

Osteoporosis
30.Patient known case of diabetes.Urine RE shows microalbuminuria.What
should be done next

ACE inhibitors

Beta blockers

Diuretics

Insulin

31. Patient came complaining of photosensitivity, malar rash, joint pain and
had RBCS in urine, what the diagnosis:

a) rheumatoid arthritis

b) lupus nephritis

c) gout

32. 45 years old female came to ER with acutely swollen knee + ballotment
patella. The most important to do

a) MRI of the knee

b) Aspiration

c) Complete blood count

d) Rheumatoid factor

33. Pathophysiology of crohn's disease:

a) Environmental toxin

b) Bacterial toxin

c) Unknown

d)Tuberculous granuloma
34. One scenario of patient 20 years old complain of bone and joint pain,
bleeding ,recurrent infection (positive myloperoxidase and prominent blast
cell)

1- myeloblastic leukemia

2-chronic myelogenous leukemia

3-mylodysplastic syndromes

35. Old patient came to your clinic for follow up . she notice that she has pain
on

her foots , hands .On examination ; Joints are swollen , tender on touch , red.
What is your diagnosis

a) Rheumatoid Arthritis

b) Anklysiong spondylaitis

c) Osteoarthritis
IN THE NAME OF ALLAH

my moh exam held on 11of may 2015

Qs was cleare and stright for word with no more detailes than that help
you to reach the right answer ,it was familier and most of it based on Qs
in facebook group so strongly irecommend to be in touch with those
groups which help you alt and directed you to right way so be in touch
with this .and of course exam need your basic knowledge so books like
masterboard and family medicine will help you in that ..wish to all of
you good lock

1]antipsychatric drug not adminsterated parentally in emergency in


psychatric presentation is:

A}ziprasidone

B}haloperidole

C}lorazepam

2]pt presented with enlarged painful bilateral knee joints there is


normal ESR and CRP and there is no signs of effusion so your diagnosis
is:

1}gout

2}rheumatoid arthritis

3}osteoarthritis

3]20yrs old lady presented in the first stage of labor, of the following
which type of anesthesia is most approprciated for her

A}morphpine
B}general anesthesia

C}local anesthesia

D}IM ......

4]best drugs used for chronic pain is:

A}ibuprofen

B}asprin

C}acetoaminophine

D}nepraxen

5]40 yrs old lady diagnosed with TB presented co red painful eyes with
photophobia diagnosis is:

A]uveitis

B]bacterial congectivitis

c]viral congectivis

D]glucoma

6]case of ENT pt working in nosey area raise up the radio because he


can not listen what this is deafness

MY ANSWER NOSEY DEFEANESS

7]clear case of OTOTIS MEDIA

8]pt on OCP for the last 2wks presented with thrombosed veins what is
the mechanism:

a]L vedin
b]DIC

c]vonwillbrand disease

d]haemophilia B

9]what is the mechanism of nose defeaness

a]sensory.

b]conductive

10]child presented with migretray arthritis +sore throat+fever what is


the diagnosis

a]rheumatic fever

b]rheumatoid arthritis

c]juvinal RA

11]case of atrial fibrillation

12]young man he stand near to box office falldown for minutes and
then recover completely diagnosis:

a]TIA

b]silent heart attack

c]fainting

13]known case of HTN on beta blocker and hydrochrothaziades


presanted with syncobal attacks he has been brought to ER
PR64,BP120/80, ECG normal pt dischared to home again he approciated
the same syncobal attacks with stable vital signs and normal ECG this is
time what would you do:
a]stop hydrochrothiazides

b]holtter monitor

c]stress ECG

14]ECG of atrial firilation diagnosis

15]old pt with past history of stroke presented with abdominal


pain,diagnosis

a]diverticolitis

b]mesentric ishaemia

16]patient on asntipsychotic treatment he has been improved the risk


of suidide

a]increased

b]decreased

c]no change

17]clear case of dehydration in infant with depressed anterior


fontanelle ,decreased tears what is your manadement:

a]ORS

b]IV rehydration

18]very obese patient with BMI42 ,he is HTN,and with painful ruptured
thrombosed veins he started on conservative management but still
hypertensive BP150/100,and painful thrombosed ruptured veins what is
your management

a]add antihypertesive
b]surgical referral

19]case of DKA what is the most likely cause:

a]isulin mismanagement

b]diet mismanagement

20]obese diabetic pt ,faliure of conservative management so you will


add :

a]insulin

b]bigunide

c]glucagon

d]thiazolidinediones

answer is C for its favourable side effect of wt reduction

21]picture of wart on foot pt is basket ball player ttt

a]elecrotransction

b]cuttarage

c]cyrotherapy

22]UTI in lady ttt

a]ciprofloxacin and check within 2 days

b]ampicilin and check within 2days

c]piperacillin/tazobactam

23]pt underwnt C/S ,co fever and vaginal discharge ttt

a]clindamyocin and gentamyocin


b]metronidazole

24]case of addision disease presented with weakness ,clammy skin..ect


ttt

a]0.9NSover 1hour+hydrocotesion IV

b]0.9 NS over 8hours+hydrocortesion IV

c]0.9 NSover 1 hr+fludrocortison orally

d]0.9NS over 8hrs +fludrocortison orally

25]HIV pt presented withpurprish shik lesion and on the mouth ttt

a]topical antibiotic

b]oral ABS

c] oral cortesion

d radiotherapy and chemotherapy

26]chlid presented with vesicals over the roof of mouth but spares the
gingiva diagnosis

a]aphsus ulcer

b]congenital ulcer

c]herpangina

27]pt ate out side presented with hepatoslenomegally ,fever after 10


days with rash over abdomen diagnosis

a]malaria

b]hepatitis
c]kalazar

d]typhoid fever

28]clear case of mursmus

29]which one of the following cause bleeding when it interact with oral
anticoagulants

a]spinach

b]garicle

c]gingo

30]pt of oligoovulation[as mentioned]due to polysystic overy syndrome


she is G2P2 was on progestron but now she stoped it befor 5 yrs this is
make her more prone to

a]cervical hyperplasia

b]breast ca

c]endometrosis

d]hip fracture

31]14 yrs old female ,menarch before 6 months presented with


menstrual cycle every 3 wks your action

if pregency test is negative it is normal

32]Q about risk of OCP vs patch of contraceptive[dermal]

dermal patch increase risk of thrmbosis more than OCP

33]diabetic pt prseneted with drooping right eyewith normal gaze the


eye is out and downwith limited up and down movementand he can not
addut the eye so the diagnosis is

a]right fasical palsy

b]right oculomotor palsy

34]pt fear from parks and zoo and she can not go out of home

a]generaized anexity disorder

b]special phobia

c]post tramatic stress disorder

35]pt diagnosed with bronchial carcinoma come for follow up the best
modulity of follow up is

a]MRI

B]CT

36]clear case of COPD

37]smoker diagnosied with broncheicisis which of the following os more


indicated

a]continous ABs

b] postural drainage

38]child presented with limp X ray showed avascular necrosis ttt

a]physical

b]surgical

c]no wt bareing

39]asymptomatic pt presented with normal ca and phosphate with


elevated ALP your ttt

a]vit D

b] ca

c]steriod

40]Dexa scan showed 3.5 diagnosis is

a]osteoprosis

b]osteopenia

c]osteomalacia

41]16 yrs old pt presented with weakness and abdominal pain he has
family history of similar condtion in his auntcolonscopy showed polyps
of adenomaso screening every

a]6months

b]3months

c]1yrs

d]10yrs

42]30 yrs old pt no medical or eye problems so eye screening should be


every

a]yearly after age of 30

b]2years till age of 50

c]50-59 twice

43]pt presented with post partum haemorragic for maternal asthma


which of the following drug is contraindicated
a]prostaglandinF2

b]oxytocin

c]progestrone

44]pt on antituberculs presented with double vision the cause is

A]INH

b]streptomyocin

c]rifampcin

45]breast self examination should be

a]zigzag

b]circular

46]pt presented with hoarsness and sternal chest pain diagnisis

a]IHD

b]GERD

47]pt with heart burn ttt

a]antiacids

48]child presented with vomitting and decrease wt all exam normal


mechanism

a]decrease perstalis in the lower oesphageal sphinter

b]mechanical

49]least likely to be associated with chronic fatigibilty syndrom

a]gradual onset
b]mild grade fever

c]for 6 months

d]pain releifedwith rest

50]child presented with lower limb tender papule with abdominal pain
diagnosis

a]ITP

b]honshon purpra

51]toxcity with acetoaminophine the first step is

a]gastric lavage

b]measure level of acetoaminophin

c]adminstration of N acteylsystein

52]metalic poisoning pt presented withhaematemisis and N/V the most


likely metal is

a] iron

b]mercury

c[lead

d]zinic

53]pt presented with S.O.b X ray showed pleural effusion with protein
less than 30mg /dl and decreased LDH so the cause of pleural effusion is

a]bronchopeumonia

b]heart faliure
c] ca...

54]pt presented with sudden onset of chest pain and S.O.B the best
investigation is

a]2 D CT

b]CT angiography

c]V/Q

55]case of burn the skin red ,blenshed the degree of burn is

a]predromal

b]1 degree

c]second degree

d] third degree

56]pt presented with frequency andrecurrent UTIinvestigations showed


bilateral enlarged kidenys with cyst with smooth barenchema so
diagnosios is

a]renal calculi

b]poly cystic kideny disease

57]43years old pt diagnosed befor 4years with rheumatiod arthritis


presented with bone loss he disapponited this is condition is due to

a]cells in synovial fluids

58]pt felt from thrid stories presented with sever brain injury the most
important survival process is

a] give prophalyatic antiepileptic


b]scure airway

c]control bp

59]case of statistics about specifity

60]case of statistics about type of study which was cross section all this
presented in umm alqura revise it well..

61]pt had fear of automobile now when he see it he had sweating


diagnosis

a]panic attack

b]special phobia

exam QS was 70 and sorry that all I can remebered ,i neither can not
remember all choices but I mentioned what is maily related to
case,revise all this is cases with each other ..all luck to you and donot
forget me of your preyers
IN THE NAME OF ALLAH

my moh exam held on 11of may 2015

Qs was cleare and stright for word with no more detailes than that
help you to reach the right answer ,it was familier and most of it
based on Qs in facebook group so strongly irecommend to be in
touch with those groups which help you alt and directed you to
right way so be in touch with this .and of course exam need your
basic knowledge so books like masterboard and family medicine
will help you in that ..wish to all of you good lock

1]an*psychatric drug not adminsterated parentally in emergency


in psychatric presenta*on is:

A}ziprasidone

B}haloperidole

C}lorazepam

2]pt presented with enlarged painful bilateral knee joints there is


normal ESR and CRP and there is no signs of e4usion so your
diagnosis is:

1}gout

2}rheumatoid arthri*s

3}osteoarthri*s

3]20yrs old lady presented in the 6rst stage of labor, of the


following which type of anesthesia is most approprciated for her

A}morphpine
B}general anesthesia

C}local anesthesia

D}IM ......

4]best drugs used for chronic pain is:

A}ibuprofen

B}asprin

C}acetoaminophine

D}nepraxen

5]40 yrs old lady diagnosed with TB presented co red painful eyes
with photophobia diagnosis is:

A]uvei*s

B]bacterial congec*vi*s

c]viral congec*vis

D]glucoma

6]case of ENT pt working in nosey area raise up the radio because


he can not listen what this is deafness

MY ANSWER NOSEY DEFEANESS

7]clear case of OTOTIS MEDIA

8]pt on OCP for the last 2wks presented with thrombosed veins
what is the mechanism:

a]L vedin
b]DIC

c]vonwillbrand disease

d]haemophilia B

9]what is the mechanism of nose defeaness

a]sensory.

b]conduc*ve

10]child presented with migretray arthri*s +sore throat+fever


what is the diagnosis

a]rheuma*c fever

b]rheumatoid arthri*s

c]juvinal RA

11]case of atrial 6brilla*on

12]young man he stand near to box oAce falldown for minutes


and then recover completely diagnosis:

a]TIA

b]silent heart aBack

c]fain*ng

13]known case of HTN on beta blocker and hydrochrothaziades


presanted with syncobal aBacks he has been brought to ER
PR64,BP120/80, ECG normal pt dischared to home again he
approciated the same syncobal aBacks with stable vital signs and
normal ECG this is *me what would you do:
a]stop hydrochrothiazides

b]holBer monitor

c]stress ECG

14]ECG of atrial 6rila*on diagnosis

15]old pt with past history of stroke presented with abdominal


pain,diagnosis

a]diver*coli*s

b]mesentric ishaemia

16]pa*ent on asn*psycho*c treatment he has been improved the


risk of suidide

a]increased

b]decreased

c]no change

17]clear case of dehydra*on in infant with depressed anterior


fontanelle ,decreased tears what is your manadement:

a]ORS

b]IV rehydra*on

18]very obese pa*ent with BMI42 ,he is HTN,and with painful


ruptured thrombosed veins he started on conserva*ve
management but s*ll hypertensive BP150/100,and painful
thrombosed ruptured veins what is your management

a]add an*hypertesive
b]surgical referral

19]case of DKA what is the most likely cause:

a]isulin mismanagement

b]diet mismanagement

20]obese diabe*c pt ,faliure of conserva*ve management so you


will add :

a]insulin

b]bigunide

c]glucagon

d]thiazolidinediones

answer is C for its favourable side e4ect of wt reduc*on

21]picture of wart on foot pt is basket ball player Bt

a]elecrotransc*on

b]cuBarage

c]cyrotherapy

22]UTI in lady Bt

a]ciproHoxacin and check within 2 days

b]ampicilin and check within 2days

c]piperacillin/tazobactam

23]pt underwnt C/S ,co fever and vaginal discharge Bt


a]clindamyocin and gentamyocin

b]metronidazole

24]case of addision disease presented with weakness ,clammy


skin..ect Bt

a]0.9NSover 1hour+hydrocotesion IV

b]0.9 NS over 8hours+hydrocortesion IV

c]0.9 NSover 1 hr+Hudrocor*son orally

d]0.9NS over 8hrs +Hudrocor*son orally

25]HIV pt presented withpurprish shik lesion and on the mouth Bt

a]topical an*bio*c

b]oral ABS

c] oral cortesion

d radiotherapy and chemotherapy

26]chlid presented with vesicals over the roof of mouth but spares
the gingiva diagnosis

a]aphsus ulcer

b]congenital ulcer

c]herpangina

27]pt ate out side presented with hepatoslenomegally ,fever aIer


10 days with rash over abdomen diagnosis

a]malaria
b]hepa**s

c]kalazar

d]typhoid fever

28]clear case of mursmus

29]which one of the following cause bleeding when it interact


with oral an*coagulants

a]spinach

b]garicle

c]gingo

30]pt of oligoovula*on[as men*oned]due to polysys*c overy


syndrome she is G2P2 was on progestron but now she stoped it
befor 5 yrs this is make her more prone to

a]cervical hyperplasia

b]breast ca

c]endometrosis

d]hip fracture

31]14 yrs old female ,menarch before 6 months presented with


menstrual cycle every 3 wks your ac*on

if pregency test is nega*ve it is normal

32]Q about risk of OCP vs patch of contracep*ve[dermal]

dermal patch increase risk of thrmbosis more than OCP


33]diabe*c pt prseneted with drooping right eyewith normal gaze
the eye is out and downwith limited up and down movementand
he can not addut the eye so the diagnosis is

a]right fasical palsy

b]right oculomotor palsy

34]pt fear from parks and zoo and she can not go out of home

a]generaized anexity disorder

b]special phobia

c]post trama*c stress disorder

35]pt diagnosed with bronchial carcinoma come for follow up the


best modulity of follow up is

a]MRI

B]CT

36]clear case of COPD

37]smoker diagnosied with broncheicisis which of the following os


more indicated

a]con*nous ABs

b] postural drainage

38]child presented with limp X ray showed avascular necrosis Bt

a]physical

b]surgical
c]no wt bareing

39]asymptoma*c pt presented with normal ca and phosphate


with elevated ALP your Bt

a]vit D

b] ca

c]steriod

40]Dexa scan showed 3.5 diagnosis is

a]osteoprosis

b]osteopenia

c]osteomalacia

41]16 yrs old pt presented with weakness and abdominal pain he


has family history of similar cond*on in his auntcolonscopy
showed polyps of adenomaso screening every

a]6months

b]3months

c]1yrs

d]10yrs

42]30 yrs old pt no medical or eye problems so eye screening


should be every

a]yearly aIer age of 30

b]2years *ll age of 50


c]50-59 twice

43]pt presented with post partum haemorragic for maternal


asthma which of the following drug is contraindicated

a]prostaglandinF2

b]oxytocin

c]progestrone

44]pt on an*tuberculs presented with double vision the cause is

A]INH

b]streptomyocin

c]rifampcin

45]breast self examina*on should be

a]zigzag

b]circular

46]pt presented with hoarsness and sternal chest pain diagnisis

a]IHD

b]GERD

47]pt with heart burn Bt

a]an*acids

48]child presented with vomiMng and decrease wt all exam


normal mechanism
a]decrease perstalis in the lower oesphageal sphinter

b]mechanical

49]least likely to be associated with chronic fa*gibilty syndrom

a]gradual onset

b]mild grade fever

c]for 6 months

d]pain releifedwith rest

50]child presented with lower limb tender papule with abdominal


pain diagnosis

a]ITP

b]honshon purpra

51]toxcity with acetoaminophine the 6rst step is

a]gastric lavage

b]measure level of acetoaminophin

c]adminstra*on of N acteylsystein

52]metalic poisoning pt presented withhaematemisis and N/V the


most likely metal is

a] iron

b]mercury

c[lead
d]zinic

53]pt presented with S.O.b X ray showed pleural e4usion with


protein less than 30mg /dl and decreased LDH so the cause of
pleural e4usion is

a]bronchopeumonia

b]heart faliure

c] ca...

54]pt presented with sudden onset of chest pain and S.O.B the
best inves*ga*on is

a]2 D CT

b]CT angiography

c]V/Q

55]case of burn the skin red ,blenshed the degree of burn is

a]predromal

b]1 degree

c]second degree

d] third degree

56]pt presented with frequency andrecurrent UTIinves*ga*ons


showed bilateral enlarged kidenys with cyst with smooth
barenchema so diagnosios is

a]renal calculi
b]poly cys*c kideny disease

57]43years old pt diagnosed befor 4years with rheuma*od


arthri*s presented with bone loss he disapponited this is
condi*on is due to

a]cells in synovial Huids

58]pt felt from thrid stories presented with sever brain injury the
most important survival process is

a] give prophalya*c an*epilep*c

b]scure airway

c]control bp

59]case of sta*s*cs about speci6ty

60]case of sta*s*cs about type of study which was cross sec*on


all this presented in umm alqura revise it well..

61]pt had fear of automobile now when he see it he had swea*ng


diagnosis

a]panic aBack

b]special phobia

exam QS was 70 and sorry that all I can remebered ,i neither can
not remember all choices but I men*oned what is maily related to
case,revise all this is cases with each other ..all luck to you and
donot forget me of your preyers
PROMETRIC EXAM 23/11/2016



1. young man, newly divorced, he ate too much and become fat, he always say no taste or
smell. All his routine investigations are normal, all brain images are normal. What is the
cause for loss of taste and smell sensations?

A. Malyringing
B. Aneurysm
C. Meningioma
D. Liver caricinoma


2. best and rapid management for specific anxiety?
A. Benzodiazipines
B. Sertraline
C. Imipramine
D. Bupropion


3. patient with major depression on SSRI for one week, presents with no improvements in
his mood. What is your action?
A. Increase the dose
B. Change medication
C. ECT
D. Follow after 3 weeks


4. patient diagnosed with depression on SSRI presented after week with some
improvements in her mood, what is your action?
A. Add another SSRI
B. Change to another class of antipsychotic
C. ECT
D. Follow up after 3 weeks


5. 30 years old male presented with following behavior: he is covering the TV because he
believes that govt is spyig him and God talks to him through the lamp. Diagnosis
A. Schizophrenia
B. Bipolar disorder
C. Mania
D. Depression



6. Douchen muscular dystrophy, asking about other ix you will do rather CPK and muscle
biopsy
A. AST
B. ESR
C. RF
D. ANA
E. Genetic testing


7. Duchenne muscular dystrophy percentage of recurrence
A. 50%
B. 25%
C. 2%
D. 7%


8. question about cyanide poisoning



9. common viral infections associated with CHF during pregnancy:
A. measles
B. herpes
C. rubella
D. chicken pox


10. anyone of the following viral infection you will treat immediately if pregnant itsmother
infected

A. hep B
B. hep C
C. CMV
D. HPV


11. pt with migraine, now she is pregnant, stopped alcohol, and smoking after getting
pregnant. Her symptoms improved with pregnancy, what is the cause/
A. Biofeedback
B. No smoking
C. No alcohol
D. Pregnancy hormones




12. term pregnant lady in delivery room, cervix dilated 8cm, station +2, CTG showed late
deceleration. Your action will be?
A. Forceps
B. Ventose
C. Spontaneous vaginal
D. CS



13. pt known case IDDM, presented with epigastric pain, polyyrea, acetone smell, what is
cause of this condition:
A. Diet mismanagement
B. Insulin mismanagement
C. Liver failure


14. pt with DM on glicazide, RBS 200, HbA1c 8, the mechanism of action of drug:
A. Increase insulin secretion
B. Increase insulin sensitivity
C. Decrease gluconeogenesis
D. Decrease carbohydrate metabolism


15. 9 year old IDDM on isulin 30/15, he take his insulin at 7 am, 7 pm. His RBS reading as
follows
SAT 11
Sun 10
Mon 9
Tues 11
Wed 10
The rest of schedule shows normal measures during the day and before sleeping. What is
this phenomenon

A. Somogyi
B. Down
C. Honeymoon


16. pt with COPD presented with recurrent episodes of dyspnea. His SPO2 92% at room air.
What is your action:
A. Penicillin
B. Salbutamol
C. Theophylline
D. Atrovent


17. 40 yrs old male presented with symptoms of DM ( polyuria, polyphagia, polydipsia) BMI
30, FBS 180, RBS 200, HbA1c 8.6, what drug you will start first:
A. Gliciazide
B. Metformin
C. Insulin
D. Acarobase



18. you study two groups with Prostrate BP, you want to see the effect of Increase weight
on BPH. Both groups are 1000 participants.
1st group with BPH you find 50 persons with increase weight
2nd group you found , 10 persons without BPH
A. OR 5
B. OR 15
C. RR 5
D. RR15


19. Elderly female with pain in right iliac fossa. On exam there is mass. She is febrile. What is
your diagnosis:
A. Acute diverticulitis
B. Crohons disease
C. Pregnancy


20. old patient with internal hemorrhoids. Her CBC show s iron deficiency anemia. What is
next to do:
A. Stool occult blood
B. US
C. Colonoscopy


21. old pt with ruptured bollus on leg, histopath shows subderaml lysis. Image attached

A. Pemphigus vulgaris
B. Empidermolysis bullosa
C. Bullous pamphigoid


22. female pt presented with 2nd degree burn in her face and neck. What is you mx:

A. Apply sulfadiazine cream, iv fluids, topical antibiotics and covrer
B. Dressing wih NIS, sulfadiazine cream and cover
C. Admit pt, sulfadiazine cream, Vaseline guaze


23. elderly male with heaviness in Rt iliac fossa. By palpastion there is no tender mass. He is
afebrile and stable, while waiting for lab results and radiology what is your action:
A. Call for general surgeon
B. Admit of evaluation
C. Send home


24. 40 yrs old multipara, she did tubal ligation 2 yrs ago after CS delivery. She has painful
nodular breast increase with menstrual cycle, she claims she eats chocolate and drink milk
during menstrual day. What is your mx:
A. OCP
B. Decrease chocolate and milk intake
C. Mammography


25. 21 yrs old female presented with painless lump in lt upper quadrant of breast. O/E firm
rubbery and mobile, no axillary lymph nodes, no skin changes, no nipple discharge, no
changes with menstrual cycle:
A. Lymphadenitis
B. Ca breast
C. Fibroadenoma


26. 55 yrs old female presented with painful axillary mass, this mass is hot and tender 5cm
in size, now pt is febrile, what is your mx:
A. Excision and drainage under GA
B. Excision and drainage under LA
C. Antibiotics and PAracetamol


27. elderly man asking for meningitis prophylaxis because his son diagnosed with
meningitis:
A. IV penicillin
B. Oral erythromycin
C. Oral rifampicin 600 mg for 4 days
D. Doxycycline


28. pt with recurrent submandibular swelling increase with chewing of food, what to do for
diagnosis:
A. Xray Neck
B. CT neck
C. FNAC



29. 42 yrs old female with 5 days postcoital vaginal discharge, white in color, no smell, no
itching. She changed her work as receptionist because of dermatitis as she used to sit for
long time on chair. What you will advise:
A. To do postcoital vaginal dousch
B. Ask about latex allergy
C. To put topical antifungal
D. To advise for antibiotics





30. what is the best eye exam for school children:
A. Fundoscope
B. Retinoscope
C. Snellen chart
D. Cover and uncover test


31. child presented with red inflamed auditory canal and pain when you pull tragus, normal
tympanic membrane. Your MX:
A. Oral penicillin
B. IV gentamycin
C. Topical antiboitcs
D. analgesia



32. A farmer presented with heaviness and dull pain in right hypochondruim, CT abdomen
shows well defined cystic mass in rt lobe of live. What is your advise:
A. pasturised milk
B. cooked meat
C. boiling water use
D. avoid handling with dogs


33. same scenario as above with same ct, pt afebrile, stable vitals with normal ix. What is
your diagnosis:

A. amoebic liver abcess
B. hydatid liver disease
C. hepatocellular Ca
D. liver cirrhosis


34. elderly male known HPT/HTN presented with spleenomegally otherwise stable, they
take biopsy from spleen the punctured side continue to bleed. His CBC shows Hb 19,
platelet 5000, what is your diagnosis?
A. Polycythemia??? Rubravera
B. Spleenomegally
C. Abdominal aneurysm


35. which of those pt you will not admit with pneumonia:
A. 60 yrs old, T 39C, RR, 30/MIN, PR 160/MIN, BP 80/60
B. 50YRS OLD, T 40C, RR35/MIN, PR 140, BP 80/60
C. 80YRS OLD T37.4C, BP 160/100, RR 20/ MIN, PR 100/MIN
D. 80 YRS OLD, TEMP 39.5 C, RR 30/MIN, BP 130/70


36. 19 yrs old male sexually active, presented with sudden and severe pain in right testis
with scrotal swelling, no penile discharge, vitally stable, all routine ix normal. What is next
step in mx:
A. start IV antibiotics
B. start oral antibiotics
C. Doppler US for testis
D. Start analgesia


36. elederly male, chronic heavy smoker, presented with heavy hematuria, rest all normal,
what you will do?
A. Renal US
B. Cystoscopy
C. CT abdomen


37. what is the first cancer you will exclude in heavy smokers:
A. Lung Ca
B. Bladder Ca
C. Colon Ca


37. elderly female presented with chest pain, diagnosed as posterior MI, then she presented
with palpitation. What is your diagnosis:

A. SVT
B. AF
C. HEART BLOCK




38. Elderly female known case of HTN/HPT presented with MI symptoms and Palpitation,
your MX:
A. Propranolol
B. Adenosine
C. Nifidipine


39. child with upper respiratory infection, presented with ear pain. O/E red tympanic
membrane, Febrile. Your first step in MX will be:
A. Antibioitcs for 10 days
B. Analgesics
C. Tympanometry
D. Gormet tube insetion




40. Child with Hx of URTI before 2 week now he is febrile ,vomiting O/E there is
RIF mass 2*3 cm what is the best imaging study to confirm diagnosis :
A. Abd CT
B. Abd U/S
C. Barium enema


41. Blood sugar in DM type 1 best controlled by :
A. short acting insulin
B. long acting insulin
C. intermediate insulin
D. Hypoglycemic agents
E. Basal and bolus insulin


Prometric 29/12/2016 drimran823@gmail.com

1. A female patient presented with oligomenorrhea, she had 3 periods in the


last year. She also had acne &
hirsutism. Her body weight was 60 kg. PV examination was normal. The
diagnosis is:
A. Polycystic ovary disease
B. Hyperprolactinemia
C. Adrenal tumor
D. Hypothyroidism
E. Premature ovarian failure

2. Female pt came with history of 4 tablets Metformin ingestion. What kind of


metabolic changes you are expecting:
A. Metabolic acidosis
B. Lactic acidosis
C. Hypoglycaemia
D. Uremia

3. Epidemic of scarlet fever, child suffering, what prophylactic medication to


the sibling who is allergic to that medicine what other medication you can
give as prophylaxis
A. Metronidazole
B. Methotrexate
C. Erythromycin
D. Doxycycline

4. 10 years old had an episode of rheumatic fever without any defect to the
heart. The
patient need to take the antibiotic prophylaxis for how long:
A. 6 months
B. 6 years
C. 15 years
D. 3 month
Prometric 29/12/2016 drimran823@gmail.com

5. 7 years old child brought by his anxious father, he concerned about


his child early puberty signs, i.e getting beard, deepening of voice and
height as compared to his age fellows. How you will describe this patient
early puberty:
A. Hypogonadism
B. Precocious Puberty
C. Late puberty
D. Delayed puberty

6. 26 years old female complaints of amenorrhea 10 weeks, lethargy


and tiredness with vomiting and nausea, having history of right
salpingits7 years ago, her menstrual history is mostly normal except
few irregular periods since menarche, she is sexually active otherwisw.
she went for her this complain to he physician he reassure these
symptoms will resolve. Upon arrival she look lethargic, temp 37.8C, BP
88/55mmgHg, PR 120/min, RR 22/min Abbomen, bowel sounds
present, generalized tenderness, no rebound tenderness, no mass felt,
no bleeding per vagina, right adenexal mass felt. Rest exam normal.
What is next appropriate step to diagnose:
A. Urine culture
B. Beta HcG
C. Progesterone trial
D. Urine dipstick

7. Young patiented with sever chest pain on right 2 hours ago, mild
dyspnea, no fever, no cough, O/E vitals normal, Chest on auscultation
Reduced Air Entry on Right later side, no murmur. XRAY attached
showing Right pneumothorax: Diagnosis:
A. Pneumonia
B. Pleurisy
C. Pneumothorax
D. Pericarditis
Prometric 29/12/2016 drimran823@gmail.com

8. Patient ate outside, presented with hepatosplenomegally ,fever and


rash over abdomen after 10 days after that trip.
Diagnosis?
A. malaria
B. hepatitis
C. kalazar
D. typhoid fever

10. 26 years old male, no known illness, presented with complaints of


white color uretheral discharge, joint pains, mild fever since 2 days,
having history of unprotected penoveginal intercourse 7 days ago with
his girlfriend, image attached, what is diagnosis:

A. Gonorrhea
B. Chlamydia
C. Fungal infection
D. E.coli infection
Prometric 29/12/2016 drimran823@gmail.com

11. 75 years old male known case of HTN, otherwise healthy, no past
significant hospital admission, presented with painful joints of hands
and feet when he wake up from sleep, and movement, O/E tender
small joints, painful on touch and little bit warm, DX

A. Osteoarthritis
B. Rheumatoid arthritis
C. Gout
D. Ankylosing spondylitis

12. 26 years old female primigravida, full term presented with labor pain,
she is in first stage of labor, what is analgesia of choice

A. IV morphine
B. Epidural anesthesia
C. General anesthesia
D. S/C morphine

12. 35 years old female K/c DM type 2, came with history of


accidently taken 4 tablets Metformin 1000mg , what metabolic
acidosis you will expect

A. Lactic acidosis
B. Metabolic acidosis
C. Hypoglycemia
D. Hyperglycemia
Prometric 29/12/2016 drimran823@gmail.com

14. Old man known case of DM , HTN on medication complain of


syncope
when he playing with his grandson associated with sweating . It is rapid
onset and
rapid recovery .His daughter said that her father completely normal
regarding his
mental and behavior status . Past history of medical admission couple
ofmonths
under indication of shortness of breath / chest pain which was
completely normal .
On examination : Vital signs are stable including normal BP, Ejection
systolic
murmur over left sternal border :

A. Decrease dose of antihyptensive to 5 mg ( it was one drugs of


diuretics and the dose is 10 mg ) as well as DM medication
B. Admit to hospital to cardiac series / investigation
C. Order Immediate ECG
D. Reassure him that this syncope due to effect of DM on autonomic
nervous

15. Patient with 2 week history cough , mild fever . On CXR : round
shadow with
Cresentric shape around it, h/o blood tinged sputum and cough “ :
A. TB
B. Aspirgelloma
C. Brachochatesis
D. Absecess
Prometric 29/12/2016 drimran823@gmail.com

16. A diabetic patient presented with exudates from a wound in his leg
with poor healing and no
sign of inflammation. What is the cause of the poor wound healing in this
diabetes meilitus patient?
A. Decreased phagocytosis process.
B. Stimulated bacterial growth.
C. Decreased immunity.
D. Increased blood supply to the wound

17. A middle age male heavy alcoholic presented at night after drink of
one bottle alcohol, with severe chest pain radiating to back, not resolving
in any position, vitals BP 110/70mmHg, PR 125/min shallow breathing,
Abdomen soft non tender, ECG normal, XRAY taken image attached
(same as in exam). What is your next appropriate step:

A. NG tube aspiration
B. Refer to surgery for immediate action
C. Barrium anema
D. Chest tube insertion
Prometric 29/12/2016 drimran823@gmail.com

18. Young male pt having only complaint of gross hematuria


otherwise normal , on examination normal , on investigation US
normal ,urine culture normal ,, now whats investigation of
choice
A. RENAL BIOPSY
B. URINE ANALYSIS
C. CYSTOSCOPY
D. RENAL ANGIOGRAPHY

A young boy presented with jaundice, high liver enzymes and kayser-
fleisher rings. what is the most proper treatment?
a) British anti-lewisite
b) Penicillamine
c) Desferroxamine

24 year old patient with asymptomatic congenital inguinal hernia:


a) Immediate surgery
b) Surgery indicated when he is >35 y
c) Elective surgery if it is reducible

Patient with idiopathic anovulation , what drug to give :


a) Clomiphene
b) Progesteron
c) LH
d) FSH

RH-ve female deliver a healthy child previosly now she is pregnant


,herhusband RH+ve , the incidence of deliver RH-ve baby :
A-0%
B-25%
C-50%
D-100%
Prometric 29/12/2016 drimran823@gmail.com







A postpartum woman complains of passage of flatus and stool through
the vagina.
What is the diagnosis?
(A) Anal fistula.
(B) Rectovaginal fistula.
(C) Ureterovaginal Fistula.
(D) Vesicovaginal fistula.


60 years old male diagnose to have acute pancreatitis, what is the


appropriatenutrition?
a)TPN
b) Regular diet with low sugar
c) High protein ,high ca , low sugar
d) Naso-jujenal tube

Treatment of Antidepressant-Induced Sexual Dysfunction SSRI:


A. Propranolol
B. Cyproheptadine
C. Fluminazel
D. Ca Carbonate
Prometric 29/12/2016 drimran823@gmail.com

30 years old patient known case of Sickle cell disease, having complaints
of recurrent pain over right hypochondrium, been diagnosed as
Cholethiasis having multiple stones in gall bladder bigger one is 3mm.
currently asymptomatic came for check up, there is no obstructive signs,
what is next choice of tx you will suggest to pt:
A. Cholecystectomy
B. Hhydrooxyurea
C. ERCP
D. Do nothing

Pregnant lady 10 weeks gestation came for checkup, asymptomatic,


having history of DVT leg in her last pregnancy pureperium, what is your
choice for her current pregnancy:
A. Start heparin immediately
B. INR followed by warfarin
C. Duplex followed by warfarin
D. Observe till DVT appear
Prometric 29/12/2016 drimran823@gmail.com

A patient have tender, redness nodule on lacrimal duct site. Before


referred him to ophthalmologist what you will do
A. Oral antibiotics
B. Nothing
C. Topical antibiotics
D. Topical steroid

There was a picture of Fused Labia Majora with some whitish secretion
i could see. The Karyotype was 46XX. USG showed intact internal
Organs. O/E Gonads Could not be found. What is the diagnosis?
A. Male Pseudohermaphroditism
B. Female Pseudohermaphroditism
C. True Pseudohermaphroditism
D. Gonadal Agenesis

A baby presented with excessive crying and blood in the stool. By


investigations abdominal x-ray shows obstructive pattern, looks like
intussusceptions. What is the management?
(A) Immediate surgical treatment.
(B) Elective surgical treatment.
(C) Barium enema.
(D) Observation.
Prometric 29/12/2016 drimran823@gmail.com

A 6 years old female brought by her parent with hematuria, all the
following investigations are needed EXCEPT:
A. Cystoscopy
B. HbS
C. Hb electrophoresis.
D. Urine analysis.
E. U/S of the abdomen to see any changes in the glomeruli.

A 28-year-old previously healthy female presents with a 2-day history of


a painful mass on her vulva. On examination vital
signs are normal. The patient is unable to sit normally because of the
pain. She has an exquisitely tender red mass in the
posterior right labia majora. It is oval in shape, with the largest dimension
being about 6 cm.
A. Bartholian cyst
B. Hemorrhoids
C. Pilonoidal sinus

Which of the following inherited blood disorders is associated with


increased bleeding time and deficiency of VIIIc:
A. hemophilia A
B. hemophilia B
C. hemophilia C
D. von willibrand disease
Prometric 29/12/2016 drimran823@gmail.com

Bipolar disorder maintenance drug

A. Lithium
B. Sodium valproate
C. Amitryptaline
D. SSRI

Child having nasal discharge from left nostril, foul smelling, and bloody,
for 6
weeks.
A) Foreign body
B) Polyp
C) maxillary sinusitis

A diabetic patient presented with exudates from a wound in his leg with
poor healing and no sign of inflammation. What is the cause of the poor
wound healing in this diabetes meilitus patient?
(A) Decreased phagocytosis process.
(B) Stimulated bacterial growth.
(C) Decreased immunity.
(D) Increased blood supply to the wound.

Female presented with large arm abcess..Managment :


A. Topical antibiotics.
B. Incision and drainage followed by antibiotics
C. Analgesics
Prometric 29/12/2016 drimran823@gmail.com


2 weeks infant came for routine check up the doctor exam the baby and he
looks well , but when the doctor ask the mother about her baby she told
somthing else she said the baby is not well he is confused and he has evil
power or somthing like that,What does the mother have?
A. Postpartum psychosis
B. Schizophrenia
C. Major Depressive disorder
D. Delusions

Man want to travel to some country which is endemic of


onchocerciasis ,he stays there for 1 week .Chances to get this
disease is:
A -HIGH
B-SEVERE
C-MINIMUM
D-NON EXISTANT

PT WITH RECURRENT NEPHROLITHIASIS ( Ca oxalate stones ) labs


show upper limit normal of Ca level in the blood + increase of Ca in the urine,
parthyroid and thyoroid all are normal, Renal function normal. What is your
advise for this patient:
A- decrease Ca intake
B- decrease protein intake
C- increase water intake
D- increase Vitamin D dose
Prometric 29/12/2016 drimran823@gmail.com

75 years old male known case of COPD since long time on medications,
presented with complaints of good sleep at daytime and but no sleep at all in
night. Sleep study shows severe hypoxemia during REM sleep. What will be
your next appropriate action to minimize his symptoms:
A. O2 concentrator at bed time
B. Positive pressure ventilation (BIPAP)
C. Tonsillectomy
D. Tracheostomy

Middle age newly diagnosed COPD, he is chronic smoker, what is your advice to
Increase the survival in this COPD patient :
a. Continues oxygen
b. inhaled bronchodilator
c. steroid
d. smoking cessation
Prometric 29/12/2016 drimran823@gmail.com

picture of a face with red scaly lesions on the nasal folds and
around the mouth, and the question is asking about the diagnosis: ( pic
almost similar to this Pic)

A. Atopic dermatitis
B. Seborrhic dermatitis
C. Psoariasis
D. Ance vulguris

What is the best way for bone and muscle to prevent aging process.
A. Low resistence excercises with conditioning
B. Low weight and conditional exercise
C. High weight and exercise.
D. Increase walk and high muscle exercise.
Prometric 29/12/2016 drimran823@gmail.com

Banker gen anxiety


1. Levothyroxine 0.25mg
2. Mass in abdomen

3. Phobia chest pain


4. Polyp in adult

5. Allergic rhinitis

6. Maryngitis
7. Sudden vision loss massage
8. Croup child
9. Diabetic retinopathy
Pediatric

1. Scenario about a child with night terror “Wake up crying, and forget in the next day”..What to
do
a) Reassure the parents
b) Referral for psychotherapy

2. 6 years old with HBsAg his mother has HBV he did not receive any vaccination except BCG he
should take:
a) DPT, HBV,MMR,OPV
b) DPT,HBV,MMR,OPV, Hib
d) Td, HBV,MMR,OPV
e) Td, HBV,MMR,OPV, Hib

3. A scenario about an infant with signs of sever dehydration. What to give?


a) Intensive ORS
b) I.V rehydration

4. Young patient, he has sickle cell anemia, He complains of fever, rash, and persistent joint pain.
Diagnosis is
a) Still’s disease “Juvenile Rheumatoid arthritis” – my answer
b) Rheumatic fever

Gyn & Obest


5. Female HIV positive, First Pap smear is negative, when to repeat?
a) 6 months
b) 12 months

6. A case about a young female discovered to have osteoporosis, What to give


c) Calcium
d) Vitamin D
e) Biphosphonate
f) All the above

7. Pregnant on iron supplementation throughout her pregnancy for her anemia, now she comes
complaining of weakness and easy fatigability Her Hemoglobin 7, MCV 60, what is the
diagnosis? 

a) Iron deficiency Anemia
b) Hypothyroidism
c) Vitamin B12 deficiency
d) Beta thalassemia

‫ﻻ ﺗﻧﺳوﻧﺎ ﻣن ﺻﺎﻟﺢ دﻋﺎﺋﻛم ﺑظﮭر ﻏﯾب‬


8. Scenario about young female with a breast mass, what is the best investigation?
a) mammogram
b) excisional biopsy
c) FNA “My answer”
d) breast US
e) follow up in 6 months

9. Female with postpartum hemorrhage after one week of delivery “Delayed hemorrhage”
,Cause of hemorrhage

g) DIC “My answer”

NB: This is a case of secondary postpartum hemorrhage “Delayed bleeding after 24 hours and less
than 12 weeks”, if “Retained placenta” option present choose it as this is the most common case for
2ry type. Uterine Atony is the most common cause for PPH in general and primary type

10. Pregnant female has a neighbor child with viral infection. Which is more dangerous for her?
a) Rubella....My answer
b) Varicella
c) Measles

11. Scenario about female with symptoms of polycystic ovary PCO, she has endometrial changes.
The question about the cause of this endometrial change?
a. Androgenic changes ….My answer
b. Forget other options!!!

12. Adult female considering pregnancy asking about varicella vaccine, what to tell her?
a) It is safe in the first trimester
b) Will not affect pregnant woman
c) It is advised to wait 1-3 months before getting pregnant after receiving it
d) It is a live attenuated Bactria

13. Female patient with a positive PAP smear, what to do next?


a) Colposcopy
b) Repeat pap smear
c) Cone biopsy

‫ﻻ ﺗﻧﺳوﻧﺎ ﻣن ﺻﺎﻟﺢ دﻋﺎﺋﻛم ﺑظﮭر ﻏﯾب‬


Chronic Diseases
14. A patient with COPD, suffering from night cough at the REM stage , what to give him?
a) Salbutamol
b) Forget other options?

15. Scenario about a case with symptoms of Asthma, What to give?


a) Start Albuterol

16. A patient with Asthma on SABA and Corticosteroids with no improvement, what to give next?
a) LABA

17. Female patient with high glucose level, she is on healthy diet since 8 week. Blood
investigations are given with high Ha1c and fasting blood sugar. What to do?
a) Continue diet for another 8 weeks
b) Start monotherapy for diabetes
c) Start Insulin

Psychiatry
18. Which one of the following below is at more risk to commit suicide?
a) 20 year college boy who had big conflict with his girlfriend
b) 60 years man who is 2 weeks on antidepressant and newly diagnosed to have osteoporosis
c) Old male I don’t remember, he was sick but not that to commit suicide.

19. Suicide Questionnaire which is wrong


a) Increase risk of Suicide “My answer”

20. Long scenario about a Patient on depression prescribed TCA at night, After taking the
medication he complained of morning vertigo and dizziness, How to deal?
a) Decrease dose of TCA
b) Shift to SSRI
c) Give TCA at morning

21. Male patient thinks that there are aliens in the backyard. He knows this is impossible but he
can’t remove the idea from his head. What is the problem?
a) Compulsion
b) Obsession
c) Delusion
d) Hallucinations

‫ﻻ ﺗﻧﺳوﻧﺎ ﻣن ﺻﺎﻟﺢ دﻋﺎﺋﻛم ﺑظﮭر ﻏﯾب‬


Dermatology
22. A case about Tenia capitis with photo

23. A case about Tenia cruris with photo

24. A case about Scabies with photo

Eye & ENT

25. Male patient with sudden painless loss of vision, Fundoscopy examination showed cherry red
macula and cloudy retina, Diagnosis
a) Retinal vein occlusion
b) Retinal Artery occlusion

26. Female patient, 16 years old, with dysphagia only on swallowing her saliva only, otherwise
normal “No fever”, Diagnosis
a) Quinsy
b) Croup
c) Globus pharynges “My answer”
d) Tonsillitis

27. Patient complain of epistaxis on cold weather, Normal blood pressure, and other
investigations are normal, What is your advice
a) Vitamin C
b) Antihestaminic
c) Anticongestant
d) Humidify air of the room

28. A scenario about a case with allergic rhinitis, what to give?


a) Anti-hestamininc
b) Anit-congestant
c) Corticosteroid
d) NSAID

29. Patient has complete ptosis in his right eye. Pupil is out and down, fixed dilated. Restricted
ocular
movements. dx
a) 3rd cranial nerve palsy.
b) 4th cranial nerve palsy.
c) 3rd and 4th.
d) 6th cranial nerve palsy

‫ﻻ ﺗﻧﺳوﻧﺎ ﻣن ﺻﺎﻟﺢ دﻋﺎﺋﻛم ﺑظﮭر ﻏﯾب‬


30. Old patients complaining of Vertigo especially on tilting his head, Dix- Hallpike maneuver is
positive, what is the treatment?
e) Epley’s maneuver “My answer”

Medical Emergencies

31. Patient with sudden severe occipital headache came to emergency. CT image is shown
a) Subarachnoid Hemorrhage
b) Intracerebral Hemorrhage
c) Meningitis

32. Old female that has been absent from work for 2 days, after that she found fallen at house,
she was thirst and drowsy with high calcium level. What to give her first?
a) Rehydration “My answer”

33. Patient with magnesium sulfate toxicity has a shallow breathing. Antidote is
a) Calcium gloconate
b) Saline
c) Sodium bicarb…My answer

34. A case of old patient with constipation and abdominal pain located at the left iliac fossa, On
examination there is palpable mass at the left iliac fossa, Diagnosis
a. Appendicitis
b. Crhon’s disease
c. Diverticulitis….My Answer

35. A scenario about an alcoholic addict. The question about the time of appearance of
withdrawal symptoms
Symptoms of Alcohol Withdrawal Syndrome

TIME OF APPEARANCE AFTER


SYMPTOMS CESSATION OF ALCOHOL USE

Minor withdrawal symptoms: insomnia, tremulousness, mild anxiety, gastrointestinal upset, 6 to 12 hours
headache, diaphoresis, palpitations, anorexia

Alcoholic hallucinosis: visual, auditory, or tactile hallucinations 12 to 24 hours*

Withdrawal seizures: generalized tonic-clonic seizures 24 to 48 hours†

Alcohol withdrawal delirium (delirium tremens): hallucinations (predominately visual), 48 to 72 hours‡ - Peak at 5 days
disorientation, tachycardia, hypertension, low-grade fever, agitation, diaphoresis

‫ﻻ ﺗﻧﺳوﻧﺎ ﻣن ﺻﺎﻟﺢ دﻋﺎﺋﻛم ﺑظﮭر ﻏﯾب‬


*— Symptoms generally resolve within 48 hours.
†— Symptoms reported as early as two hours after cessation.
‡— Symptoms peak at five days.

36. Antidote for opioid toxicity


a) Naloxone

37. 12 yrs old complain of LL, UL and face edema and other cardiac sym. Dx :
a) Wet beriberi
b) Dry beriberi
c) Vit. A deficiency

38. Patient with red swelling at the back increases in size


a) Cavernous hemangioma

39. Scenario about young female with cyanosed finger and tingling at cold weather, Diagnosis
a) Raynaud’s disease

40. A scenario about group of people stayed at a hotel, and then next day they developed cough,
diarrhea, and abdominal pain. What is your action?
a) Good ventilation + Air sanitation
b) Good ventilation + Air flow control
c) Water sanitation
d) Water flow control

41. Patient infected with a single stranded flavivirus.


a) Heb A
b) Heb B
c) Heb C
d) Heb D

42. Female patient, complain of syncopal attacks. She has past history of Aortic stenosis.
Otherwise normal. What is the cause of the syncope?
a) Hypotention
b) Valvular rupture

43. Old patient on Hydrochlorothiazide, complaining of syncope. All investigations were normal.
ECG is negative. What to do next?
a) Stop hydrochlorothiazide
b) Holter monitor….My answer

‫ﻻ ﺗﻧﺳوﻧﺎ ﻣن ﺻﺎﻟﺢ دﻋﺎﺋﻛم ﺑظﮭر ﻏﯾب‬


44. A scenario about a female has just received her first varicella vaccine, when to give the second
dose?
a) 2 weeks
b) 6 weeks
c) 1 month

45. Best analgesic for chronic pain is


a) Acetaminophen
b) Ibuprofen
c) Naproxen

46. Old patient with pain at shoulder, hip, and back especially when he wake up. What the
pathogenesis of this disease?
a) Wear down of the cartilage of the joint
b) Precipitation of crystals at the joint
c) Autoimmune disease that causes inflammation of the joint

47. Patient with a past history of dental procedure and received Kefalex antibiotic, After that she
developed abdominal pain and diarrhea.. With sigmoidiscopy there is hyperemia.., what to
give?
a) Vancomycine my answer, As a case of pseudomembraneous colitis
b) Clarythromycine
c) Penicillin

48. Old male with diarrhea and past history of recent antibiotic, On Sigmidoscopy there is White
plaques..Diagnosis
a) Pseudomembraneous colitis
b) Amoebic dysentery

49. A case of young female has colonoscopy which revealed polyps, interval to check again
a) 6 months
b) 1 year
c) Every 5 years

50. Young female with Hx of night sweat and wt loss for about 6 month splenomegally-reed
sternberg cells in blood picture your diagnosis is:
a) Hodgkin’s lymphoma
b) non-Hodgkin’s lymphoma
c) EBV

‫ﻻ ﺗﻧﺳوﻧﺎ ﻣن ﺻﺎﻟﺢ دﻋﺎﺋﻛم ﺑظﮭر ﻏﯾب‬


51. A case of Leukemia with blast cells:
a) AML
b) ALL “my Answer”
c) CML
d) CLL

52. Patient with a bleeding disorder due to low blood clot factor VIII and FVIII gene mutation,
Diagnosis:

a) Hemophilia A
b) Hemophilia B
c) Hemophilia C
d) Von Willebrand diseases

53. A question about Pseudohyperparathyrodism investigation

54. A question about Cushing syndrome?

‫ﻻ ﺗﻧﺳوﻧﺎ ﻣن ﺻﺎﻟﺢ دﻋﺎﺋﻛم ﺑظﮭر ﻏﯾب‬


MCQ PROMETRIC

(Surgery)
14.best treatment for tension pneumothorax & pt in distress:
a) IVF
b) O2
c) Respiratory stimulator
d) Aspiration of air by needle
e) Intubation

(ENT)
15.ranula:
a) Forked uvula
b) Thyroglossal cyst
c) Swelling at the floor of mouth
d) !!

(Medicine)
16.critical count of platelets which lead to spontaneous bleeding is:
a) 1000
b) 50.000
c) 75.000
d) 100.000
e) 200.000
N.B. less than 20.000

(Surgery)
17.which one will give bilateral breast CA:
a) lobular breast ca (ILC)
b) intraductal breast ca (IDC)
c) mucinous breast ca
d) paget disease
e) medullary breast ca
f) tubular breast ca
N.B. ILC (bilateral)- IDC (common)

(Surgery)
18.the best method for temporary control of bleeding is:
a) arterial tourniquet
b) venous tourniquet
c) direct finger pressure
d) adrenaline
e) !!

GOOD LUCK
N.B. well known method in the story of ABCDE

GOOD LUCK
(Psychiatry)
19.anorexia nervosa, all true except:
a) lethargy
b) langue hair
c) amenorrhea
d) young female
e) !!
N.B. I think e will be the wrong choice

(Psychiatry)
20.hypochondriasis, all true except:
a) more common in medical students
b) less common in male
c) more common in lower social class
d) defined as morbid preoccupation of one's body or health

(ENT)
21.all are speech disorders except:
(I am not sure whether b and d is a real choices or not)
a) Stuttering
b) Mumping
c) Cluttering
d) Palilia
e) !!

N.B. Types of speech disorders

Cluttering
Stuttering
Apraxia
Lisp
Rhotacism
Spasmodic dysphonia
Aphasia
Dysarthria
Huntington's disease
Laryngeal cancer
Selective mutism
Specific Language Impairment
Speech sound disorder
Voice disorders

GOOD LUCK
(Psychiatry)
22.family behavior toward schizophrenic pt affect prognosis adversely:
a) double binding
b) over emotion behavior
c) schismatic parents
d) projective identification
e) !!
N.B. I didn't understand the question.

(Urology)
23.premature-ejaculation, all true except:
a) most common sexual disorder in males
b) uncommon in young men
c) Benefits from sexual therapy involving both partners
d) it benefit from anxiety Rx
e) !!
N.B. Premature ejaculation (PE) is the most common sexual
dysfunction in men younger than 40 years.

(ENT)
24.the most prominent symptoms of acute otitis media is:
a) Pain
b) Hearing loss
c) Discharge
d) Tinnitus
e) Non of the above
N.B. triad (pain-deafness and tinnitus)

(Anesthesia)
25.length of trachea in adult is:
a) 11-12 cm
b) 24cm
c) 20cm
d) 4cm
e) Non of the above
N.B. it is a fact that mentioned in applied anatomy book

GOOD LUCK
-

GOOD LUCK
(Surgery)
26.indication of trachostomy, all true except:
a) foreign body in larynx
b) LT recurrent nerve cut
c) CA larynx
d) In some procedure which involve in radiation exposure
e) Non of the above
N.B. bilateral recurrent N injury is an indication

(ENT)
27.fetal unilateral nasal discharge is feature of:
a) Adenoid
b) Choanal atresia
c) Foreign body
d) RT atrophy !!
e) !!
N.B. forget everything about ENT except this question. It is quite
common

(ENT)
28.best first aid to control epistaxis is:
a) Adrenaline
b) Cold application on forehead
c) Good pinching or compression lower end of nose for 5-8 min
d) Non of the above
e) !!
N.B. whenever you see bleeding, pressing is the usual intervention

(Psychiatry)
29.known risk factor of suicide include all of the following except:
a) depression
b) previous self attempt
c) females less than males
d) drug and alcohol dependence
e) if doctor ask the pt any suicidal attempt
N.B. it is well demonstrated in psychiatry book (dr.alsagir)

GOOD LUCK
)

GOOD LUCK
(Surgery)
30.gastric lavage:
a) ineffective after 12 hrs Paracemamol intake
b) indicated with paraffin oil
c) used more in semiconscious pt than induced vomiting
d) pt should be in RT side
e) !!
N.B. actually ineffective after 1 hr.

(Surgery)
31.A 50 yr old man presented with central abdominal pain radiated to
back. Abdominal and back x ray is normal. Dx is:
a) cholecystitis
b) appendicitis
c) pancreatitis
d) diverticulitis

(Medicine)
32.acute GN, all is acceptable Ix except:
a) complement
b) urinanalysis
c) ANA
d) Blood culture
e) Cystoscopy

(Medicine)
33.all of the following precipitate seizure except:
a) hypourecemia
b) hypokalemia
c) hypophosphatemia
d) hypocalcemia
e) hypoglycemia

(Medicine)
34. A 25 yr old pt presented with headache , avoidance of light & resist
flexion of neck, next step is:
a) EEG
b) C-spine X-ray
c) Phonation
d) Non of the above
e) !!
N.B. I suspect meningitis, the Rx is Abx + LP

GOOD LUCK
)

GOOD LUCK
(Medicine)
35.A 20 yr old female present with fever, loin pain & dysuria,
management include all of the following except:
a) urinanalysis and urine culture
b) blood culture
c) IVU (IVP)
d) Cotrimexazole
e) !!
N.B. I suspect PN. So, Rx includes admission, Abx & re-hydration.

(Surgery)
36.surgery- the most effective monitoring method in pt with acute
bleeding is:
a) HB
b) HCT
c) Vital sign
d) Amount of blood loss
e) !!
N.B. Blood loss could be internal one

(continue)
1 (NAA – 2009)

37- (ENT)
The most common cause of epistaxis in children is:
a) polyps
b) trauma
c) dry air
d) thrombocytopenia
e) !!

Most cases of epistaxis do not have an easily identifiable cause.


Local trauma (ie, nose picking) is the most common cause, followed by
facial trauma, foreign bodies, nasal or sinus infections, and prolonged
inhalation of dry air. A disturbance of normal nasal airflow, as occurs
in a deviated nasal septum, may also be a cause of epistaxis.
Epistaxis is more prevalent in dry climates and during cold weather.

GOOD LUCK
38- (pedia)
The amount of Na+ in ORS “oral rehydration solution” in (WHO) is:
a) 150 meq
b) 120
c)90
d)60
e) 30

Composition of standard and reduced osmolarity ORS solutions

Reduced Osmolarity ORS solutions Standard ORS solution

(mEq or mmol/l) (mEq or (mEq or


(13, 15-18, 28- mmol/l) mmol/l) (mEq or mmol/l)
29) (6, 14, 22-27) (21)

75 75-90 111 111 Glucose

75 60-70 50 90 Sodium

65 60-70 40 80 Chloride

20 20 20 20 Potassium

10 10 30 10 Citrate

245 210-260 251 311 Osmolarity

39- (pedia)
Child with epiglotitis will present with all of the following EXCEPT:
a) fever
b) dysphagia
c) like to lie in supine position
d) stridor
e) !!

Epiglottitis usually presents abruptly and rapidly with fever, sore throat,
dysphagia, respiratory distress, drooling, and anxiety.
Physical: Patients tend to appear seriously ill and apprehensive.
Characteristically, patients have a "hot potato" muffled voice and may
have stridor. Usually children will assume the "sniffing position" with
their nose pointed superiorly to maintain an adequate airway.

GOOD LUCK
40- (medicine, heam)
The likelihood of a daughter for father having severe hemophilia B is:
a) 0 %
b) 25 %
c) 50 %
d) 75 %
e) 100 %

GOOD LUCK
involves a lack of the clotting facyor IX . (sex linked, X linked recssive)
Men and women each have 23 pairs of chromosomes. Women have two X
chromosomes; men have one X and one Y chromosome. Hemophilia is an X-
linked genetic disorder, which means that it's passed from mother to son on
the X chromosome. If the mother carries the gene for hemophilia on one of
her X chromosomes, each of her sons will have a 50% chance of having
hemophilia.

41- (pedia)
All of the following are true about pyloric stenosis, EXCEPT:
a) incidence male more than female
b) onset is generally late in the first month of life
c) vomitus is bile stained
d) appetite is good
e) jaundice occur in association

nonbilious vomiting that increase in volume and frequency is seen>>


alkalosis >> low K+, low cl-, and metabolic alkalosis. Unconjugated
hyperbilirubinemia is also present.

42- (pedia)
Risk factor of sudden death syndrome include all of the following, EXCEPT:
a) cigarette smoking during pregnancy
b) old primigravida
c) crowded living room
d) prematurity
e) small gestational age

potential risk factors include:


smoking, drinking, or drug use during pregnancy
poor prenatal care
prematurity or low birth-weight
mothers younger than 20
smoke exposure following birth
overheating from excessive sleepwear and bedding
stomach sleeping
Stomach sleeping. Foremost among these risk factors

GOOD LUCK
43- (pedia vs. surgery)
A 2yr boy has rectal pain, bleeding with perinatal itching and constipation for
3 days, physical examination revealed a perianal erythematous rash which
extend 2 cm around the anal ring, most likely Dx:
a) anal fissure
b) rectal polyp
c) ulcerative colitis
d) streptococcal infection
e) malacoplakia

I’m not sure

44- (med, ID)


in brucellosis, all of the following are true EXCEPT:
a) brucella abortus cause more severe form than B. melitansis in children
b) human to human is rarely document
c) human can be infected through inhalation
d) brucella species are small, non motile gram –ve coccobacilli
e) pt with high titer can show false –ve

brucella abourtus is less likely to cause more severe disease in cattles &
human than b. melitansis. Rarely transmitted by breast feeding & intercourse
(human to human). Human can be infected through eating, drinking,
inhalation, skin wound. It is a gram –ve rod.

45- (pedia)
Children are expected to walk without support at age of:
a) 6 months
b) 9 monthes
c) 15 monthes
d) 18 monthes
e) 20 monthes

… (abo warda) text book

GOOD LUCK
46- (pedia)
Which of the following vaccines NOT given to a household contact with
immunodeficient child:
a) hepatitis
b) DPT
c) oral polio
d) BCG
Who should not get the oral polio vaccine? OPV should not be given when
there is a higher risk of bad effects caused by the vaccine, including the
following:
Being moderately or severely (badly) ill with or without fever.
Having someone in the house with a weak immune system.
History of a severe allergic reaction to a dose of OPV.
Long-term treatment with steroid medicine.
Weak immune system. The immune system is the part of the body that
normally fights off sickness and disease. A weak immune system may
be caused by cancer, HIV or AIDS, inborn immune deficiency, or
taking medicines, such as chemotherapy.

47- (pedia)
Symptoms of cystic fibrosis in neonate:
a) meconium ileus
b) pneumothorax
c) steatorrhea
d) rectal prolapse
e) !!

meconium ileus is associated with CF (defect in chromosome 7, autosomal


recessive)

48- (pedia)
DKA in children, all of the following are true EXCEPT:
a) don’t give K+ till lab results come
b) ECG monitoring is essential
c) if pH < 7.0 give HCO3-
d)NGT for semiconscious pt
e) furosemide for pt with oligouria

give fluid (volume resuscitation) is the goal. Polyuria is one of DKA


symptoms, not oligouria.

GOOD LUCK
)

GOOD LUCK
49- (med, onco)
Common symptoms of Hodgkin lymphoma not seen in non Hodgkin
lymphoma:
a) night sweat
b) superior vena cava syndrome
c) CNS involvement
d) intussusception
e) bone pain

I don’t know

50- (pedia)
To prevent tetanus in neonate:
a) give anti-tetanus serum to neonate
b)give immunoglobulin to mother
c) give tetanus toxoid
d) give antibiotics to mother
e) give penicillin to child to kill tetanus bacilli

DTP= diphtheria, tetanus & pertusses


D&T are toxoids, P is inactivated bacteria
Route: IM

51- (pedia)
MMR given at age of:
a) 3 months
b) 8 months
c) 12 months
d) 24 months
e) !!!

52- (pedia)
Hypothyroid in young baby usually due to:
a) endocrine irresponse
b) enzyme def.
c) drug by mother
d) agenesis
c) !!!

maldescent of the thyroid and athyrosis are the commonest cause of sporadic
congenital hypothyroidism … (abo warda)… but not realy sure

GOOD LUCK
)

GOOD LUCK
53- (med)
Blood pressure, all of the following are true EXCEPT:
a) if 2/3 of cuff false high BP
b) internal cuff must cover 80% of arm
c) follow circadian vary late night high BP
d) high BP 3 standard deviation away from normal
e) you have to have more than one reading to Dx high BP

a. ?, b. ?, c. no circadian variation in BP, d. ?, e. more than one reading will


Dx H BP.

54- (med)
All of the following drugs advised to be given to elderly pt, EXCEPT:
a) cimitidine
b) thyroxin
c) digoxin
d) chloropromide
e) !!

is a sulphonylurea, best avoided in elderly ppl and in those with renal failure.

55- (surgery)
Percentage (%) of reinfarction for pt undergoing non-cardiac surgery:
a) 5%, 3 months after the infarct
b) 15% , 3 months after the infarct
c) 35%, 3 months after the infarct
d) 5%, 3-6 months after the infarct
e) 35%, 3-6 months after the infarct

Reinfarct risk upon undergoing a non-cardiac surgery:-


<3 months after MI 37% of patients will reinfarct
3-6 months after MI 15%
>6 months after MI risk remains constant at 5%
reinfarct carries a 50% mortality rate

56- (med)
Furosemide increase excretion of :
a) Na+
b) K+
c) phosph.
d) non of the above

GOOD LUCK
e) !!

GOOD LUCK
furosemide causes high blood Na+, urea, glucose, cholesterol. And low blood
K+, Ca+.

57- (med)
Heparin anticoagulant action depend on :
a) potentiation of antithrombin three
b) change plasmin to plasminogen
c) affect prothrombin
d) affect ionized Ca++
e) !!!

heparin potentiates antithrombotic affect in antithrombin three


warfrin inhibits vitamin K-dependent gamma carboxylation of factors 2, 7,
9, 10.

58- (clinical pharmacology)


Digoxin toxicity :
a) tinnitus
b) plural effusion
c) nausea
d) all of the above
e) non of the above

Extracardiac symptoms: Central nervous system: Drowsiness, lethargy,


fatigue, neuralgia, headache, dizziness, and confusion may occur.
Ophthalmic: Visual aberration often is an early indication of digitalis
toxicity. Yellow-green distortion is most common, but red, brown, blue, and
white also occur. Drug intoxication also may cause snowy vision,
photophobia, photopsia, and decreased visual acuity. GI: In acute and
chronic toxicity, anorexia, nausea, vomiting, abdominal pain, and diarrhea
may occur. Mesenteric ischemia is a rare complication of rapid intravenous
infusion. Many extracardiac toxic manifestations of cardiac glycosides are
mediated neurally by chemoreceptors in the area postrema of the medulla.
Cardiac symptoms: Palpitations, Shortness of breath, Syncope, Swelling of
lower extremities, Bradycardia, Hypotension

GOOD LUCK
59- (community med)
Communicable diseases controlled by:
a) control the source of infection
b) block the causal of transmition
c) protect the susciptable pt
d) all of the above
e) non of the above

60- (OB/Gyn)
Anti D ig not given to a pregnant if :
a) 25- 28 wk
b) anti D Ab titer of 1:8
c) after amniocentesis
d) after antepartum hemorrhage
e) after chorion villi biopsy

- Anti-D is routinely given to un-sensitized mothers at 28 and 34 wks of


gestation
- Fetomaternal haemorrhage sensitizes susceptible mothers to develop anti-D
antibodies (e.g. Birth, Miscarriage, abortion, amniocentesis, vaginal bleeding,
external cephalic version ..etc) Indications for Anti-D
- The initial response to D antigen is slow sometimes taking as long as 6
months to develop (rising titers)

61- (med)
Blood pH :
a) high after diarrhea
b) low after vomiting
c) more in Rt atrium than Lt atrium
d) lower in Rt atrium than Lt ventricle
e) lower in renal vein than renal artery

a. after diarrhea (which is alkali) the blood will be acidic (low pH)
b. after vomiting (which is acidic ”Hcl”) the blood pH will be alkali (high
pH)
c. O2 low H+ and high pH… so, the pH in Rt atrium “ low O2” will be
lower than the Lt atrium “high O2”
d. Lt ventricle has more oxygenated blood than Rt atrium
e. blood in arteries is more oxygenated than that in veins

GOOD LUCK
62- (OB/Gyne)
Premenstrual tension :
a) more in the first half of menses
b) 60% associated with edema
c) associated with eating salty food
d) menorrhagia
e) !!

Premenstrual syndrome (PMS) is a recurrent luteal phase condition (2nd


half of menses) characterized by physical, psychological, and behavioral
changes of sufficient severity to result in deterioration of interpersonal
relationships and normal activity
The most common signs and symptoms associated with premenstrual
syndrome include:
o Emotional and behavioral symptoms
Tension or anxiety
Depressed mood
Crying spells
Mood swings and irritability or anger
Appetite changes and food cravings
Trouble falling asleep (insomnia)
Social withdrawal
Poor concentration

o Physical signs and symptoms


Joint or muscle pain
Headache
Fatigue
Weight gain from fluid retention
Abdominal bloating
Breast tenderness
Acne flare-ups
Constipation or diarrhea
One study has shown that women with PMS typically consume more dairy
products, refined sugar, and high-sodium foods than women without
PMS. Therefore, avoidance of salt, caffeine, alcohol, chocolate, and/or
simple carbohydrates may improve symptoms.

GOOD LUCK
63- (OB/Gyne)
Blockage of first stage labor pain by :
a) block of the lumbosacral plexus afferent
b) block of the lumbosacral plexus efferent
c) block of the pudendal nerve
d) block of sacral plexus
e) !!

I don’t know

64- (OB/Gyne)
If a pregnant eating well balanced diet, one of the following should be
supplied :
a) Ca++
b) phosph.
c) vit. C
d) none of the above
e) !!

I don’t know

65- (OB/Gyne)
Most important cause of immediate post partum hemorrhage :
a) laceration of cervix
b) laceration of vagin
c) uterine atony
d) placental fragment trtention

all of these choices are true but the commenest cause of PPH is uterine atony.

66- (OB/Gyne)
Dysparunea caused by all of the following EXCEPT :
a) cervicitis
b) endometriosis
c) lack of lubricant
d) vaginitis
e) uterine prolapse

Sx of uterine prolapse are: feeling of haviness of fullness in the pelvis,


backache, perulent discharge, decubitus ulceration, bleeding.

GOOD LUCK
GOOD LUCK
67- (OB/Gyne)
All of the following are normal flora and should not treated, EXCEPT:
a) trichomonus
b) candida
c) E.coli
d) fragmented bacteria
e) !!

trichomonus vaginalis is a STD, caused by a flagellated bacteria, Rxed by


metronidazole, causes green yellowish discharge, male partner should be
Rxed as well.

68- (OB/Gyne)
Rx of bacterial vaginitis
a) ampicillin
b) tetracycline
c) metronidazol
d) erythromycin
e) !!

69- (ER)
management of anaphylactic shock all of the following, EXCEPT :
a) I.V.F
b) 100% O2
c) corticosteroid
d) !!
e) !!!

Management of anaphylaxis is summarized by:-


Epinephrine + Diphenhydramine, then oxygen + IV fluids

70- (surgery)
All of the following are signs of allergy to local anesthesia, EXCEPT :
a) laryngeal spasm
b) urticaria
c) low BP
d) bronchospasm

I don’t know

GOOD LUCK
)

GOOD LUCK
71- (Surgery)
gastric aspiration :
a) cuffed NGT may prevent aspiration
b) !!
c) !!!

72- (Surgery)
Below the inguinal ligament, where is the femoral artery :
a) medial
b) lateral
c) anterior
d) posterior
e) !!

from medial to lateral vein, artery, nerve

GOOD LUCK
GOOD LUCK
2010

(Medicine)
73- Hepatitis most commonly transferred by blood is:
a) HBV.
b) HAV.
c) HCV (previously known as non a non b).
d) None of the above.

Answer: c. HBV transmission by blood was common before effective


screening tests and vaccine were available. HAV is transmitted via enteral
route. HCV recently with PCR technology began to have a screening test, but
transmission remains high ass many infected individuals are carriers.

(Medicine)
74- Primary TB:
a) Usually involves upper lobe of lung.
b) Normal X-ray.
c) +ve PPD test.
d) None of the above.
e) All of the above.

Answer: c. Primary TB has some x-ray findings, although non-specific. X-


ray is normal only in 15% of pts with primary TB. Has a +ve PPD test and is
characterized by lower lobe disease.

(Medicine)
75- Increased bleeding time is seen in all of the following except:
a) Hemophilia.
b) Scurvy.
c) VwD (Von-Willebrand disease).

Answer: a.

GOOD LUCK
(Medicine)
76- Serum ferritin reflects:
a) Total iron stores.
b) Serum iron.
c) Bone marrow iron.
d) None of the above.

Answer: a. Serum iron is reflected by TIBC which is an indirect measure of


transferrin.

(Medicine)
77- Which one shifts oxyheamoglobin dissociation curve to the LEFT:
a) Hypoxia.
b) Acidosis.
c) High altitude.
d) None of the above.

Answer: a. Hypoxia leads to releasing myoglobin from muscles leading to left


curve. Acidosis and high altitude both cause right shift. NOTE: Alkalosis
also shifts the curve to the left.

(Community Medicine)
78- Treatment of contacts is applied in all of the following except:
a) Bilharisiasis.
b) Malaria.
c) Hook worm.
d) Filariasis.

Answer: a. Bilharisiasis (schistosomiasis) is transmitted by exposure to fresh


water inhabitited by cercariae stage of the species that are expelled by snails
infected by it. So, prevention is by avoiding these areas. However for
malaria and filariasis, the vector lives in close contact with humans
(mosquitoes). And hook worm larvae live in the soil contaminated by feaces
of infected humans.

(Community Medicine)
79- The best way for health education:
a) Mass media.
b) Interview.
c) ??

GOOD LUCK
Answer: I don’t know

GOOD LUCK
(Clinical Pharmacology)
80- Which one of these drugs is administered orally:
a) Amikacin.
b) Neomycin.
c) Gentamycin.
d) Streptomycin.
e) Tobramycin.

Answer: b. All aminoglycosides have very poor oral uptake. Neomycin is too
toxic for enteral use. So it is given orally (also Kanamycin) mainly to act on
bowel flora in preparation for bowel surgery.

(Clinical Pharmacology)
81- Chronic use of steroids will give:
a) Osteomalacia.
b) Myopathies of pelvic girdle.
c) Increased risk of breast Ca.
d) Hypoglycemia.

Answer: b. Steroids will cause osteoporosis by inhibiting Vit. D, not


osteomalacia. There has been no association with breast Ca. It causes
hyperglycemia and steroid-induced diabetes. Steroids will cause proximal
myopathy.

(ENT)
82- Swallowed foreign body will be found in all of the following except:
a) Stomach.
b) Tonsil.
c) Pharyngeal pouch.
d) Piriform fossa

Answer: b.

(Medicine)
83- All of the following are true about pulmonary embolism, except:
a) Normal ABG.
b) Sinus tachycardia is the most common ECG finding.
c) Low plasma D-dimer is highly predictive for excluding PE.
d) Spiral CT is the investigation of choice for diagnosis.
e) Heparin should be given to all pts with high clinical suspicion of PE.

GOOD LUCK
Answer: a. In PE, ABG will show decreased PaO2 and PaCO2.

GOOD LUCK
(Clinical Pharmacology)
84- All of the following cause gastric irritation, except:
a) Erythromycin.
b) NSAIDS.
c) Sucralfate.
d) Diclofenac.
e) Penicillins.

Answer: c. Penicillins cause gastric irritation if given orally at high doses.


(Clinical Pharmacology)
85- All of the following are anti-arrhythmic drugs, except:
a) Xylocaine.
b) Digoxin.
c) Quinidine.
d) Amiodarone.
e) Procainamide.
Answer: a. Lidocaine (not xylocaine) is the local anesthetic that is also an
anti-arrhythmic.
(Pediatrics)
86- Apgar score:
a) Heart rate is an important criterion.
b) Is out of 12 points.
c) Gives idea about favoribilty of vaginal delivery.
d) Taken at delivery time and repeated after 5 minutes.
e) Respiratory rate is an important criterion.

Answer: a. Apgar score reflects condition and well-being of infant at birth. It


is out of 10 points, and is taken 1 minute after delivery then repeated at
5minutes after delivery. Heart rate and respiratory effort (not rate) are the
most important criteria. Other criteria are color, reflex irritability and muscle
tone.
(Urology)
87- Old male came with urine retention, dilated ureter and hydronephrosis,
Dx is:
a) Benign prostatic hyperplasia.
b) Ureteric stone impaction.
c) Cladder tumor.

GOOD LUCK
Answer: a.

GOOD LUCK
(Medicine)
88- In DKA, use:
a) Short and intermediate acting insulin.
b) Long acting insulin.

Answer: a. Short acting insulin is most preffered to avoid causing


hypoglycemia. Also important measures in treatment of DKA are fluid and
potassium replacement along for searching for a source of infection and
treating it.

(Urology)
89- In Testicular torsion, all of the following are true, except:
a) Very tender and progressive swelling.
b) More common in young males.
c) There is hematuria.
d) Treatment is surgical.
e) Has to be restored within 12 hours or the testis will infarct.

Answer: c.

(Ophthalmology)
90- Question about congenital squint

(Medicine)
91- All of the following causes secondary HTN, except:
a) Pheochrmocytoma.
b) Addison’s disease.
c) Hyperaldosteronism (conn’s disease)
d) Renal disease.
e) Pregnancy.
f) Primary hypothyroidism.

Answer: b. Addison’s disease causes postural hypotension. Pregnancy


induced HTN occurs in pre-eclampsia. And yes, Primary hypothyroidism as
well as thyrotoxicosis cause secondary HTN.

GOOD LUCK
GOOD LUCK
(Clinical Pharmacology)
92- All of the following are true about paracetamol poisoning, except:
a) Metabolic acidosis.
b) Hypoglycemia.
c) Bronchospasm.
d) Liver Failure.
e) Acute renal tubular necrosis.

Answer: c.

(ENT)
93- Adenoids:
a) Can be a chronic source of infection.
b) Causes snoring.
c) Located at the back of the nasopharynx 1 inch above the uvula.
d) Involved in the immune system reaction.
e) All of the above.

Answer: e.

(Pediatrics)
94- Cellulitis in children (6 – 24 months) is most commonly caused by:
a) H. influanzae.
b) Group A Streptococcus.
c) Staphylococcus.

Answer: b. Although gram –ve organisms (such as H. influanzae) occur at a


higher rate in immunocomprimised and in children than in normal adults,
Group A strept. remains the most common causative organism of cellulitis.

GOOD LUCK
GOOD LUCK
2011

(OB/Gyne)
01 All the following drugs should be avoided in pregnancy EXCEPT:
a) Na+ Valproate.
b) Glibenclamide.
c) Keflex.
d) Septrin.
e) Warfarin.
Answer = b) Glibenclamide.

(Community Medicine)
02 Secondary prevention is best effective in:
a) DM.
b) Leukemia.
c) Pre-eclampsia.
d) Malabsorption.
Answer = d) Malabsorption.

(Surgery)
03 Complications of colostomy are all the following EXCEPT:
a) Malabsorption of water.
b) Prolapse.
c) Retraction.
d) Obstruction.
e) Excoriation of skin.
Answer = e) Excoriation of skin.

- 186 -

GOOD LUCK
(

GOOD LUCK
(Medicine)
04 Regarding rubella infection, one is TRUE:
a) Incubation period is 3-5 days.
b) Causes oral ulcers.
c) Causes arthritis.
d) Does not cause any heart problem to the fetus.
Answer = c) Causes arthritis.

(Orthopedics)
05 Avascular necrosis of the head of femur is usually detected clinically
by:
a) 3 months.
b) 6 months.
c) 11 months.
d) 15 months.
Answer = a) 3 months. X-rays are diagnostic in the first few weeks. No
source I reviewed talked about when it can be 'clinically' detected.

(Ophthalmology)
06 All the following may cause sudden unilateral blindness EXCEPT:
a) Retinitis pigmentosa.
b) Retrobulbar neuritis.
c) Retinal detachment.
d) Vitreous hemorrhage.
e) Central retinal artery embolism.
Answer = a) Retinitis pigmentosa. It causes gradual night blindness.

(Medicine)
07 Fecal leukocytes come with all EXCEPT:
a) Shigellosis.
b) Clindamycin induced colitis.
c) Idiopathic ulcerative colitis.

GOOD LUCK
Answer = b) Clindamycin induced colitis.

GOOD LUCK
(Medicine)
08 In a child with TB, all is found EXCEPT:
a) A history of exposure to a TB patient.
b) Chest x-rays findings.
c) Splenomegaly.
d) A (+ve) culture from gastric lavage.
Answer = All is correct!

(Medicine)
09 In brucellosis, all is true EXCEPT:
a) Back pain.
b) Hepatomegaly.
c) Splenomegaly.
d) Lymphadenopathy.
e) Gastroenteritis.
Answer = e) Gastroenteritis.

(Medicine)
10 All can be used for the treatment of acute gout EXCEPT:
a) Allopurinol.
b) Penicillamine.
c) Gold salt.
d) Paracetamol.
e) Indomethacin.
Answer = b) Penicillamine.

(Pediatrics)
11 In a 6 months old patient with sepsis, the most likely organism will be:
a) Listeria.
b) ‫ﺁ‬-Hemolytic Streptococci.
c) H. Influenza type B.
d) Staph. Epidermis.
Answer = c) H. Influenza type B.

GOOD LUCK
(Medicine)
12 In mycoplasma pneumonia, there will be:
a) A (+ve) cold agglutinin titer.
b) Lobar consolidation.
Answer = Both are correct!.. A positive cold agglutinin titer occurs in
50-70% of patients, and lobar consolidation may also be present but rare.

(Medicine)
13 The treatment of community acquired pneumonia is:
a) First generation cephalosporin.
b) Penicillin G + second generation cephalosporin.
c) Erythromycin.
d) Erythromycin + Gentamycin.
Answer = c) Erythromycin.

(Pediatrics)
14 All are vaccines given in Saudi Arabia to normal children EXCEPT:
a) TB.
b) Pertussis.
c) H. Influenza type B (HiB).
d) Mumps.
e) Diphtheria.
Answer = All are given nowadays, HiB was the correct
answer a few years ago but not anymore.

(Pediatrics)
15 UTI in children is:
a) Diagnosed by isolation of 105 of the same organism by a clean
catch.
Answer = No other choice!

GOOD LUCK
GOOD LUCK
(Medicine)
16 Hypokalemia occurs with all EXCEPT:
a) Metabolic alkalosis.
b) Acute tubular acidosis.
c) Chronic diarrhea.
d) Hyperaldosteronism.
e) Furosemide.
Answer = b) Acute tubular acidosis.

(Medicine)
17 Urine analysis will show all EXCEPT:
a) Handling phosphate.
b) Specific gravity.
c) Concentrating capacity.
d) Protein in urine.
Answer = a) Handling phosphate.

(Medicine)
18 In acute renal failure, all is true EXCEPT:
a) Phosphatemia.
b) Uremia.
c) Acid phosphate increases.
d) K+ increases.

Answer = c) Acid phosphate increases.

(Surgery)
19 In a patient with anaphylactic shock, all are correct treatments
EXCEPT:
a) Epinephrine.
b) Hydralazine.
c) Adrenaline.
d) Aminophillin.
Answer = b) Hydralazine. It causes anaphylactic reactions.

GOOD LUCK
GOOD LUCK
(Surgery)
20 A partial thickness burn:
a) Is sensitive.
b) Is insensitive.
c) Will change to slough within 2-3 weeks.
d) Needs a split graft.
e) Needs a free flap.
Answer = Could be sensitive if superficial, insensitive if deep,
will start to heal within 2-3 weeks, and may need a split graft
if deep. So, all is true EXCEPT (e).

(Urology)
21 In an 82 years old patient with acute urinary retention, the management
is:
a) To empty the bladder by Foley's catheter and follow up in the
clinic.
b) To insert a Foley's catheter then send the patient home to come
back in the clinic.
c) To admit and investigate by TURP.
d) Immediate prostatectomy.
Answer = b) To insert a Foley's catheter then send the
patient home to come back in the clinic.

(Medicine)
22 A 6 years old female from Jizan with haematuria, all the following
investigations are needed EXCEPT:
a) Hb S.
b) Cystoscopy.
c) Hb electrophoresis.
d) Urine analysis.
e) U/S of the abdomen to see any changes in the glomeruli.
Answer = c) Hb electrophoresis.

GOOD LUCK
GOOD LUCK
(Medicine)
23 In a patient with Hb = 8, MCV = 82, retic = 10%, all is needed
EXCEPT:
a) Hb electrophoresis.
b) Coombs test.
c) Serum iron level.
d) Serum bilirubin level.
Answer = c) Serum iron level.

(Medicine)
24 A boy with a cola urine colour, 3 weeks back a throat swab showed
group A â-hemolytic streptococci, all is in favor of diagnosing post
streptococcal glomerulonephritis EXCEPT:
a) Red cell casts in urinalysis.
b) Increased creatinine.
c) Streptozyme test.
d) Decreased complements.
e) Shrunken kidney by U/S.
Answer = e) Shrunken kidney by U/S. Its either normal or slightly
enlarged.

(Medicine)
25 In a patient with weight loss, all can be a cause EXCEPT:
a) Thyrotoxicosis.
b) Nephrotic syndrome.
c) TB.
d) AIDS.
Answer = b) Nephrotic syndrome will cause edema = more weight

GOOD LUCK
GOOD LUCK
(Medicine)
26 A 36 years old female with FBS = 14 mmol & glucosuria, without
ketones in urine, the treatment is:
a) Intermittent I.M. insulin NPH.
b) Salphonylurea + diabetic diet.
c) Diabetic diet only.
d) Metformin.
Answer = c) Diabetic diet only.

(Surgery)
27 50 years old female with rectal bleeding, on examination an external
hemorrhoid was found, the treatment:
a) Advice excision of hemorrhoid.
b) Do nothing and follow up in 6 months.
c) Send home on iron tablets.
d) Bowel enema + colonoscopy.
e) Rigid sigmoidoscopy.
Answer = d) Bowel enema + colonoscopy.

(Surgery)
28 In peritonitis:
a) The patient rolls over with agony (pain).
b) The patient lies still.
c) Pulse rate is decreased.
Answer = a) The patient rolls over with agony (pain).

(OB/Gyne)
29 A cord prolapse occurs in all EXCEPT:
a) Premature rupture of membranes.
b) Preterm delivery with rupture of membranes.
c) Oligohydramnios.
d) Head high in pelvis.
Answer = c) Oligohydramnios.

GOOD LUCK
GOOD LUCK
(OB/Gyne)
30 In diabetes in pregnancy:
a) Oligohydramnios occurs.
b) Hypoglycemia occurs in the baby after delivery.
c) Hypercalcemia occurs in the baby.
Answer = b) Hypoglycemia occurs in the baby after delivery.

(Surgery)
31 Stress ulcers can be found in all EXCEPT:
a) Burns.
b) Aspirin.
c) CNS lesions.
d) Penicillin.
Answer = d) Penicillin.

(Medicine)
32 Peripheral neuropathy can occur in all EXCEPT:
a) Lead poisoning.
b) DM.
c) Gentamycin.
d) INH (anti-TB).
Answer = All can cause peripheral neuropathy!

(Medicine)
33 In a patient with upper abdominal pain, all is in favor of peptic ulcer
EXCEPT:
a) Hunger pain.
b) Heart burn.
c) Epigastric mass.
d) Epigastric tenderness.
e) Hematemesis.
Answer = c) Epigastric mass.

GOOD LUCK
(Medicine)
34 Premalignant lesions have:
a) Pedunculated polyps.
b) Villous papilloma (adenoma).
c) Polypoid polyp.
d) Juvenile polyp.
Answer = b) Villous papilloma.

(Surgery)
35 Multiple ulcers on the medial aspect of the leg with redness and
tenderness around it are most likely:
a) Venous ulcers.
b) Ischemic ulcers.
c) Carcinoma.
Answer = a) Venous ulcer.

(Surgery)
36 A 35 years old female with bloody discharge from the nipple, on
examination there is cystic swelling near areola, the most likely diagnosis is:
a) Duct ectasia.
b) Intra-ductal papilloma.
c) Fibroadenoma.
Answer = b) Intra-ductal papilloma.

GOOD LUCK
GOOD LUCK
2011

(Surgery)
37- Appendicitis most diagnostic:
a) fever
b) diarrhea
c) urinary symptoms
d) leukocytosis
e) tender Rt lower quadrant with rebound

(Surgery)
38- Pt known to have gall stones presented with central abd. Pain and
bruising in the flanks, Dx

a) acute cholecystitis
b) acute pancreatitis
c) !!
d) !!
e) !!

(Orthopedics)
39- congenital hip dislocation (CDH)

a)Dx after 3 yrs


b)abduction + flexion (ortolani test) produce click
abduction not limited
lengthening of the leg
rx by open reduction

(Orthopedics)
40- Supra-condylar fracture pt presented with swelling and cyanosis of
finger after plaster. Management:

a) Removal of splint near finger


b) Entire removal of all splint
c) !!
d) !!!

GOOD LUCK
(Orthopedics)
41- Adduction hip & internal rotation in fixed position will be :

a) Ant. Dislocation of hip


b) Post. Dislocation of hip
c) !!
d)!!!
e)!!!!

(Pediatrics)
42- Neonatal just delivered , term pregnancy. Developed resp.distress
CXR showed multicystic lesion in Lt side shifted mediastinum to the Rt ,
decreased bilatral breath sound & flat abdomen:

a) Diaphragmatic hernia
b) RDS
c) Emphysema
d) !!
e) !!!

(Pediatrics)
43- A 2 month – boy with projectile vomiting. On examination olive mass
in Rt upper quaderant of abdomen. 1st step of investigation is:

a) x-ray abd.
b) U&E
c) Barium study
d) !!
e) !!!

(Surgery)
44- 2 day-old neonate presented with peri-rectal bleeding, Dx

a) Mickles diverticulum
b) Intussception
c) Fissure –in-ano
d) !!
e) !!!

GOOD LUCK
intussception : blood PR (jam like)

GOOD LUCK
(Surgery)
45- Child with imperforated anus the most useful diagnostic procedure
is:

a) Plain X-ray of abd. with child inverted position


b) Plain X-ray abdomen
c) !!
d) !!!
e) !!!!

(Pediatrics)
46- 18 month-old pt ,the mother complain that pt is saying only mama
baba , no other words .pt otherwise completely normal. 1st step to
evaluate :

a) Physical examination
b) Hearing test
c) Developmental test
d) Test speech
e) !!

(Pediatrics)
47- perinatal asphyxia could cause by all EXCEPT :

a) Abruptio placenta
b) Hyper emisis gravidium
c) Pre-eclampsia
d) !!
e) !!!

(OB/Gyne)
48- Sign and symptoms of normal pregnancy,EXCEPT:

a) Hyperemesis
b) Hegar sign
c) Chadwick's sign
d) Amenorrhea

Hegar sign: softening of the lower uterine segment

GOOD LUCK
Chadwick's sign : bluish discoloration to the cervix and vaginal walls

GOOD LUCK
(OB/Gyne)
49- In twins all true, EXCEPT :

a) Dizygote more common than monozygote


b) In dizygote more twin-to twin transfusion
c) Physical changes double time than single form
d) U/S can show twins
e) !!

(OB/Gyne)
50- Ectopic pregnancy, EXCEPT :

a) Occur ovarian in 20%


b) Empty uterus by u/s with high beta-HCG before 12 wks
c) Beta –HCG double of normal
d) !!
e) !!!!

(OB/Gyne)
51- Breech presentation all true , EXCEPT:

a) Breech after 36 wks about 22%


b) Known to cause intra-cranial hemorrhage
c) Known with prematurity
d) !!
e) !!!

(OB/Gyne)
52- In lactation all true, EXCEPT:

a) Sucking stimulate prolactin


b) Sucking cause release of oxytocin
c) Milk release deacreased by over hydration
d) !!
e) !!!

GOOD LUCK
)

GOOD LUCK
(OB/Gyne)
53- secondary amenorrhea

a) always pathalogical
b) is part of sheehan syndrome
c) turner syndrome
d) !!
e) !!!

(OB/Gyne)
54- Pt with post partum hemorhage & infertility, all can be found
EXCEPT:

A) Balloning of sella turcica


b) Decrease Na
c) Hypoglycemia
d) Decreased T4
e) Decreased iodine uptake

no idea

(OB/Gyne)
55- Placenta previa, all true EXCEPT:

a) Shock out of proportion of bleeding


b) Malpresentation
c) Head not engaged
d) Painless bleeding
e) !!

(OB/Gyne)
56- PID(pelvic inflammatory dis), all true EXCEPT:

a) Infertility
b) Endometriosis
d) Dysparunia
c) Can be treated surgically
e ) !!!

GOOD LUCK
GOOD LUCK
(Surgery)
57- Laprascopy could be used in all, EXCEPT:

a) Infertility
b) Intestinal obstruction
c) 1ry amenorrhea
d) !!
e) !!!

(OB/Gyne)
58- Recurrent abortion:

a) Genetic abnormality
b) Uterine abnormality
c) Thyroid dysfunction
d) DM
e) Increased prolactin

(OB/Gyne)
59- DIC occur in all ,EXCEPT:

a) Abruptio placenta
b) Fetal death
c) DM
d) Pre-eclampsia
e) !!

(OB/Gyne)
60- Pregnancy induced HTN, all true EXCEPT:

a) Ankle edema
b) Polyuria
c) Exaggerated reflex
d) RUQ pain
e) !!

GOOD LUCK
GOOD LUCK
(OB/Gyne)
61- Pyelonephritis in pregnancy , all true EXCEPT:

a) Gentamycin is drug of choice


b) Abruptio placenta should ruled out
c) E .coli common organism
d) Should be treated even for asymptomatic
e) !!

(Medicine)
62- All indicate severity of bronchial asthma ,EXCEPT:

a) Intercostal and supraclavicular retraction


b) Exhaustion
c) PO2 60 mmHg
d) PO2 60 mmHg +PCO2 45 mmHg
e) Pulsus paradoxis > 20mmHg

Severe: PEFR<50% Sa O2 <91%,PCO2 >42, dyspnea at rest, inspiratory &


expiratory wheezes, accessory muscle use , pulsus paradoxus >25 mmHg

(ENT)
63- All are normal in association with teething EXCEPT:

a) Rhinorrhea
b) Diarrhea
c) Fever > 39 C
d) Irritability
E) !!

(Medicine)
64- Pt come within 3 hrs H/O Lt side weakness , examination revealed Lt
side hemiparesis, pulse 120/min irregular with diastolic murmer at mitral
area. 1st step of nanagement :

a) heparin
b) digoxim
c) EEG
d) carotid angiography

GOOD LUCK
e) echo

GOOD LUCK
(Medicine)
65- pt with fever , pallor petechei, echemosis, CBC as WBC 2,800 /mm3
,Hb 6 &
plt 2900 . next step of investigation :

a)bone marrow aspiration


b) !!
c) !!!
d) !!!

(Medicine)
66- pt on chemotherapy presented with fever , all should be done ,
EXCEPT:

a) blood culture
b) urine culture
c) aspirine is effective
d) broad spectrum antibiotics
e) !!

(Surgery)
67- crohn's disease indication of surgery is:

a) internal fistula
b) external fistula
c) intestinal obstruction
d) abd.mass ????
e) !!

surgery reserved for complication as fistulae,obstructon,abscess,perfortion&bleeding)

(Surgery)
68- in affected index finger, all can be used , EXCEPT:

a) rubber tourniguet
b) xylocaine
c) adrenalin
d) ring block
e) !!

no idea

GOOD LUCK
GOOD LUCK
(Medicine)
69- pt with low grade fever and arthalgia for 5 days, presented with
pansystolic murmer at the apex.H/O difficulty in swallowing with fever 3
wks back. Most likely diagnosis:

a)bacterial endocarditis
b) viral myocarditis
c) acute rheumatic fever
d) pericarditis
e) !!

major criteria(polyarthritis,pericarditis,chorea,eryth.margenatum
&subcutaneous nodule)

(Medicine)
70- sign of congestive heart failure in children all .EXCEPT:

a)gallop rhythm
b) periorbital edema
c) basal crept.
d) hepatomegaly
e) bounding pulse

(Medicine)
71- 7 month-old infant with 4 months H/O interruption of feeding,
normal S1 loud S2 pansystolic murmer grade III/IV at 3rd Lt intercostal
parasternally,
with hyperactive pericardium. Dx:

a) large VSD
b) large ASD
c) PDA
e) AR
e) MR

VSD(delayed growth,holosystolic murmer at LLSB


PDA(poor feeding ,bounding pulse,hyperactive pericordium,continuous
murmer
best heard infravlavicular and sometimes systolic at leftsternal edge

GOOD LUCK
GOOD LUCK
(Medicine)
72- Pt presents with fever swelling is felt,Ant.lymph node swelling warm,
tender &fluctuant Dx:

a) viral infection
b) bacterial lymphadenitis
c) Hodgkin L.
d) ALL
e) !!

GOOD LUCK
GOOD LUCK
2012

(SURGERY)
73- A 50 years old female patient with H/O weight loss, preference
of cold weather, palpitation, there is H/O firm swelling in the
anterior neck for 5 years Dx:
a) Simple goiter
b) Diffuse toxic goiter (gravis disease)
c) Toxic nodular goiter
d) Carotid body tumor
e) Parathyroid edema

Thyrotoxic symptoms: Most patients with toxic nodular goiter (TNG)


present with symptoms typical of hyperthyroidism. Symptoms include
heat intolerance, palpitations, tremor, weight loss, hunger, and
frequent bowel movements.

(SURGERY)
74- Patient presented with fluctuant redness of finger bulb.
Treatment:
a) Incision
b) Penicillin
c) !!
d) !!
e) !!

Paronychia is a soft tissue infection around a fingernail. Paronychia occurs in


2 forms: acute and chronic. The etiology, infectious agent. Treatment: If
without obvious abscess, be treated nonsurgically. If an abscess has
developed, incision and drainage must be performed.

(OB/GYN)
75- Infertility, all true, EXCEPT:
a) Male factor present 24%
b) Normal semen analysis is >20,000,000
c) Idiopathic infertility is 27%
d) High prolactin could be a cause
e) !!

GOOD LUCK
(MEDICINE)
76- Patient with H/O fever, peripheral blood film +ve for malaria:
a) Banana shaped erythrocyte is seen in P. vivax
b) Mostly duo to P. falciparium
c) Treated immediately by primaquin 10mg for 3 days
d) Response to Rx will take 72 hr to appear
e) !!

The majority of malaria infection is caused by either P. falciparum or P.


vivax, and most malaria-associated deaths are due to P. falciparum.
RBC shapes don’t change if infected with malaria. Primaquine is used
for irradication od P.ovale & p.vivax. Chloroquine is the 1st line of
treatment & is used in 2 doses.

(GYN/OBST)
77- Primary amenorrhea duo to:
a) Failure of canalization of mullarian duct
b) Kallmann syndrome
c) Agenesis
d) All of the above
e) Non of the above

Primary amenorrhea:
-No menses by age of 14 and absence of 2ry sexual CCx.
-No menses by age of 16 with presence of 2ry sexual CCx
Causes: Gonadal dysgenesis 30%, Hypothalamic-pituitary failure e.g
Kallmann syndrome( defecient GnRH), congenital absence of uterus
(20%) “agenesis of Mullerian system”, Androgen insensitivity (10%),

(SURGERY)
78- Patient oliguria one contraindicated: "??"
a) I.V. ringer lactate
b) I.V.P
c) !!
d) !!
e) !!

Q is not clear for me :s

GOOD LUCK
GOOD LUCK
(SURGERY)
79- Patient with multiple trauma, conscious.Rx:
a) ABC
b) I.V.F
c) Cross match
d) !!

(SURGERY)
80- Among the causes of Prtal HTN, which of these will cause the
lease hepatocellular damage
a) Schistosomiasis
b) Alcoholic cirrhosis
c) Post necrotic scaring
d) Cirrhosis duo to chronic active hepatitis
e) !!

(GYN/OBST)
81- Obstructed labor: "??"
a) Primigravida
b) Easy to be Dx early before starting labor
c) !!
d) !!
e) !!
Q is not clear for me :s

(SURGERY)
82- RTA with urethral bleeding. Step of management:
a) Insert foley's cath
b) Stabilize the pelvis
c) Insert suprapubic cath
d) !!
e) !!

The life is too short but… you can do something going on & everyone
can remember you in good things that you did

GOOD LUCK

You might also like